Clinical Integration Case 1 student questions

Ace your homework & exams now with Quizwiz!

A 60 year old female presents with fatigue, pain in her joints, pale skin, swollen lymph nodes and recent weight loss. When taking the patient's medical history, she informs the physician that she suffers from anemia and takes folic acid occasionally. A Complete Blood Count test was ordered, and the most significant findings were low levels of RBCs and high levels of WBCs. These findings could lead the physician to suspect what possible disease? What additional tests should be ordered to confirm this diagnosis? A. Aplastic anemia; bone marrow aspiration B. Rickets: alkaline phosphatase test C. Acute myeloid leukemia; bone marrow biopsy D. Cutaneous T-Cell lymphoma; skin biopsy E.Multiple myeloma; bone marrow biopsy

(A) INCORRECT- With aplastic anemia WBC and RBC counts are usually both low; to diagnose aplastic anemia a bone marrow biopsy is usually performed. (B) INCORRECT- Although the patient presents with bone pain, Rickets usually does not cause any changes in blood cell levels. Additionally, Rickets is observed in predominantly children; high levels of alkaline phosphatase indicate increased osteoblastic activity which confirms Rickets. (C) CORRECT- Leukemia causes increased development of immature WBCs which crowd out RBCs and eventually spill into the bloodstream leading to higher levels of WBC's and lower levels of RBC's; bone marrow biopsy is the usual test performed to diagnose leukemia. (D) INCORRECT: Cutaneous T-Cell lymphoma is attributed with a high level of WBC's but usually presents with rashes and/or scaly, raised patches on the skin. The patient does not present with these symptoms and therefore cannot be correct; A skin biopsy is usually performed to confirm T-cell lymphoma. (E) INCORRECT- Multiple myeloma usually presents with low RBCs and WBCs. If the CBC revealed abnormal light chain levels and beta-2 microglobulin levels this disease could be a possible suspicion for the physician; A bone marrow biopsy and bone marrow aspiration (along with a blood test) is used to confirm multiple myeloma.

A 24-year-old female showed up to the emergency room with fever, abdominal cramps, and dysentery. She states that the illness originally started out as watery diarrhea. She was diagnosed with Enteroinvasive E. Coli. What has allowed the bacteria to invade the colonic epithelial cells? A. Actin tails B. Chronic ulceration C. pInv genes D. Shiga toxin E. K1 capsular antigen

A. Actin tails (Incorrect. This allows the bacteria to move in the cytoplasm after the bacteria has already invaded.) B. Chronic ulceration (Incorrect. This is a result of the destruction caused by the inflammatory infiltration.) C. pInv genes (Correct. pInv genes allow the bacteria to invade the colonic epithelial cells.) D. Shiga toxin (Incorrect. Shiga toxin is found in STEC which has the subsets VTEC and EHEC). E. K1 capsular antigen (Incorrect. This allows E. Coli to pass the blood brain barrier and cause meningitis.)

The parents of a 5-year-old boy walk into their primary care office and are worried about their son. Their child keeps holding onto his stomach, indicating some type of lower abdominal cramps and when he needs to defecate, he is straining to defecate. Parents have reported seeing pus and blood in his stool. While going over his history, his parents state that they work full time through the week and he is mostly at his daycare throughout the day. What type of bacterial infection is presumed to be in this boy based on his symptoms/history? A. Salmonella typhi B. Yersinia pestis C. Shigellosis D. Klebsiella pneumonia E. Proteus mirabilis

A. A is not a correct answer because Salmonella typhi will cause typhoid fever, which is a febrile illness. There is no evidence of febrile illness in the question. B. B is not a correct answer because Yersinia pestis causes bubonic plague and pneumonic plague. Bubonic plague involves high fever and painful bubo in the groin or axilla. Pneumonic plague has symptoms of fever, malaise and pulmonary signs. The question does not display any evidence of fever or pulmonary symptoms. C. C is a correct answer because Shigella is most commonly found in day-care centers, nurseries, and custodial institutions. It is transmitted person to person. Its cardinal symptoms include lower abdominal cramps and tenesmus (straining to defecate) with abundant pus and blood in stool. This can be found on page 261. D. D is not a correct answer because Klebsiella pneumonia mostly causes community or hospital-acquired primary lobar pneumonia. Pneumonia caused by Klebsiella species frequently involves necrotic destruction of alveolar spaces, formation of cavities, and production of blood-tinged sputum. E. E is not a correct answer because Proteus mirabilis is the most common member of the Proteus genus that causes UTI. There is nowhere in the question did the patient have some type of urinary infection.

A two-year-old of Mediterranean descent presented with enlarged, distorted cheekbones and signs of growth retardation. A CBC revealed low hemoglobin, low MCV, and excess iron. Additionally, her family's medical history showed a lineage of an autosomal homozygous genetic disorder. What kind of illness does this child have? A. Beta-thalassemia major B. Sickle cell anemia C. Alpha-thalassemia minor D. Immunohemolytic anemia E. Rickets

A. Beta-thalassemia major Correct: Signs and symptoms of beta-thalassemia major occur early in a child's life and if untreated, it can lead to distorted bones and delayed growth. Someone with beta-thalassemia minor/trait would be asymptomatic or have mild anemia, so the patient in this case will not have this. Also, blood tests for beta-thalassemia minor/trait can be mixed up with an iron deficiency. Excess iron deposits can lead to secondary hemochromatosis, which is characteristic of beta-thalassemia. Checking the patient's family history is important to determine if patients have this genetic disorder. Beta-thalassemia major is caused by homozygous carriers of β+ and/or β0. Beta-thalassemia minor/trait is heterozygous (β+/β or β0/β) and anyone with this is often asymptomatic. B. Sickle cell anemia Incorrect: Sickle cell is caused by a defect in hemoglobin itself while thalassemia is associated with defects in the beta or alpha-globin gene. It also doesn't cause distorted bones while beta-thalassemia major uniquely causes distorted cheekbones and other skeletal deformities. C. Alpha-thalassemia minor Incorrect:Someone with alpha-thalassemia minor/trait would be asymptomatic or have mild anemia. Alpha and beta-thalassemia major are much more severe. For alpha-thalassemia major, any extrauterine life would not be able to survive since it cannot form normal hemoglobin. Thus, death would occur before or quickly after birth. D. Immunohemolytic anemia Incorrect: Immunohemolytic anemia is caused by antibodies, which lead to premature RBC destruction. Thus, this condition can be ruled out since there's no mention of antibodies causing hemolysis. E. Rickets Incorrect: This disease is due to a vitamin D deficiency, causing softening and weakening of the bones. Though the question mentions a child with some type of bone issue, the question does not mention those specific signs and a vitamin deficiency.

A 21-year old female presents to her university's Student Health Center with unusual tiredness and pale skin. She claims that she has been sleeping regularly and maintaining a healthy diet. Her PA believes that she may have iron deficiency and suggests taking an iron supplement. However, she returns to the Student Health Center within a month expressing the same concern. After completing a routine blood test, her PA discovers hypochromia, microcytosis, basophilic stippling, and target cells in the patient's peripheral blood smear. She has reason to believe that the patient has 𝛃-Thalassemia minor. Which of the following is diagnostically useful? A. Hemoglobin electrophoresis revealing increased HbA2 levels B. Lab test revealing a normal total iron-binding capacityC. Genetic screening indicating 𝛃-globin tetramer, HbH D. Direct Coombs antiglobulin test indicating agglutination E. Genetic screening indicating a PIGA mutation

A. Correct: A patient with 𝛃-Thalassemia minor is typically asymptomatic but may present signs that might be mistaken for iron deficiency. An increased HbA2 level is indicative of 𝛃-Thalassemia minor, which is why electrophoresis is the best diagnostic tool to use here. B. Incorrect: This test is typically used as an indicator for iron deficiency. Though this could be used to rule out iron deficiency for the patient, it is not the best diagnostic tool to use in order to confirm 𝛃-Thalassemia minor. C. Incorrect: This would be an ideal diagnostic tool for a patient with Hemoglobin H (HbH) Disease D. Incorrect: This would be a positive indicator for Immunohemolytic Anemia. E. Incorrect: This would an indicator of Paroxysmal Nocturnal Hemoglobinuria (PNH)

A man of Middle Eastern descent is admitted to the emergency room with typhoid fever. The patient is found to have an X linked disorder that results in both intravascular and extravascular hemolysis. Which of the following is not observed in the patient? A. Heinz bodies B. Oxidative stress C. Splenomegaly D. Anemia E. Hemoglobinuria

A. Heinz bodies Incorrect: For patients with G-6-P Dehydrogenase Deficiency, oxidation results in membrane precipitations called Heinz bodies. Due to these bodies, intravascular hemolysis can occur. However, typically cells illustrating these bodies are consumed by phagocytes. B. Oxidative stress Incorrect: For the patient to be experiencing a hemolytic episode, there needs to be some cause of hemolytic stress. Furthermore the case cites the patient has typhoid fever, which is an infection that can trigger a hemolytic episode . C. Splenomegaly Correct: Unlike other illnesses featuring chronic hemolysis, G-6-P Dehydrogenase Deficiency is marked by periods of recovery. Therefore the spleen is not consistently enlarged by destroyed red blood cells. D. Anemia Incorrect: Due to the hemolysis described in the case, anemia is an expected side effect. E. Hemoglobinuria Incorrect: Because the patient is experiencing oxidative stress that triggers hemolysis, some amount of red blood cells will be seen in the urine.

A 57 year old male comes to the ER with abdominal cramps and constipation. He notices blood and pus in his stool. The bacteria stained pink on a gram stain and grew as yellow colonies when plated on MacConkey agar. The bacteria is also H2S negative. What best describes the virulence of the identified bacteria? A. The bacteria adheres to the intestinal epithelium through colonization factors and produces heat stable and heat labile enterotoxins. B. The bacteria adheres to the intestinal epithelium and destroys the microvilli. C. The B subunit of the toxin binds to 28S rRNA and inhibits protein synthesis. D. The bacteria invades the M cells in the Peyer's Patch and replicates in vacuoles. E. The bacteria invades the M cells in the Peyer's Patch and replicates in the host's cytoplasm.

A. Incorrect: ETEC produces heat stable/heat labile enterotoxins and invade the small intestine through colonization factors. B. Incorrect: EPEC adheres to the intestinal epithelium and destroys the microvilli. C. Incorrect: The A subunit of EHEC binds to 28S rRNA and inhibits protein synthesis. D. Incorrect: Salmonella replicates in endocytic vacuoles after invading the M cells. E.Correct: Both salmonella and shigella invade the M cells in the Peyer's patch by the type 3 secretion proteins and replicates in the host cell's cytoplasm

A 50-year-old female presents to the clinic with abdominal pain and diarrhea that has been ongoing for 5 days. Symptoms began after a recent visit to a petting zoo. Her symptoms have progressively worsened with bloody diarrhea. She denies any recent travel or change in diet. Patient with pallor and jaundice on exam. A fecal culture was taken and grown on MacConkey Agar with pink colonies present. What toxin is associated with this pathogen? Lab results: Results Normal range Hemoglobin 9 g/dL 12-16 g/dL Hematocrit 32% 37-43% Platelet 130 x 109/L 150-400x109/L Creatinine 1.9mg/dL 0.5-1.1mg/dL A. LT1 and STa B. Enterotoxin C. Hemolysin D. Stx1 and Stx2 E. Plasmid encoded toxin

A. Incorrect: LT1 and STa are toxins associated with ETEC which is known as traveler diarrhea and characterized with watery diarrhea. B. Incorrect: Enterotoxin is present in S. Aureus and B. Cereus which are both gram positive bacteria and do not grow on MacConkey agar. C. Incorrect: Hemolysin is associated with Vibrio which is characterized with watery diarrhea and usually associated with the ingestion of shellfish. D. Correct: Stx1 and Stx2 is associated with EHEC(STEC) whose pathogen is E. coli a lactose positive gram-negative rod hence the pink colonies on MacConkey Agar. The labs also show anemia with low hemoglobin and platelet count, as well as possible kidney damage with the elevated creatinine. On exam the patient also has jaundice and pale skin which are associated with Hemolytic uremic syndrome, a complication of EHEC(STEC). E. Incorrect: Plasmid encoded Toxin is associated with EAEC (enteroaggregative E. Coli) which is associated with chronic diarrhea and growth retardation in children.

A 27 year old woman enters the Emergency Room with a fever, and non-specific complaints of a headache, constipation, and myalgia. The fever has developed over the past few days and is unresponsive to acetaminophen. She has no recent travel history, but she mentions that her mother is visiting from Brazil and arrived 12 days ago. Since arriving, her mom has cooked almost every night. The doctor samples the bacteria and concludes that this is an H2S positive bacteria that colonizes in the gallbladder. What is a trait associated with this bacteria? A. Locus of Enterocyte Effacement Pathogenicity Island B. O157:H7 C. Pathogenicity Island I and II D. Plasmid encoding pINV genes E. Plasmids that encode Fraction 1 gene and Plasminogen Activator Protease gene

A. Incorrect: Locus of enterocyte effacement is a pathogenicity island used to encode virulence factors for Enteropathogenic E. Coli. B. Incorrect: O157:H7 is the serotype Shiga Toxin-Producing E. Coli and used for identification of the disease. C. Correct: Salmonella is controlled by two large clusters of genes called Pathogenicity Islands. Specifically, Pathogenicity Island I controls salmonella-secreted invasion proteins (Ssps) and a type III secretion system while Pathogenicity Island II contains genes that allow the bacteria to evade the host's immune response and encodes a second type III secretion system for this function. D. Incorrect: pINV are a series of genes on a plasmid used to invade the colonic epithelium in Enteroinvasive E. Coli. E. Incorrect: Yersinia pestis uses these genes as virulence factors.

A 24 year old female presents with the following symptoms: watery diarrhea, abdominal cramps, and vomiting. The lab results from the cultures collected revealed the bacteria was a gram negative rod, lactose fermenter, and growth of pink colonies was observed on MacConkey Agar. These results confirmed that the pathogen was E.coli. Which of the following is the epidemiology of E.coli? A. The most common gram-negative rods isolated from patients with sepsis B. Responsible for causing more than 80% of all community-acquired UTIs C. Prominent cause of gastroenteritis D. All of the above E. A&C

A. Incorrect: Partially correct, E.coli is the most common gram negative rods isolated from patients with sepsis (illustrated in figure 25-3 with Escherica at 45% for incidence of sepsis) however more than one answer is applicable (Page 255-256). B. Incorrect: Partially correct, see correct answer for further explanation (page 255) C. Incorrect: Partially correct, see correct answer for further explanation (page 255) D. CORRECT: E. coli's success as a pathogen is due to it being (1) the most common bacteria found in patients with sepsis, (2) responsible for causing more than 80% of all community-acquired UTIs as well as many hospital acquired infections, and (3) a prominent cause of gastroenteritis (page 255). E. Incorrect: Although E.coli is the most common gram negative rods isolated from patients with sepsis and is the prominent cause of gastroenteritis, another answer is applicable (page 255).

A 37 year-old female presents to the emergency department with a 2-day history of fever, nausea, vomiting, and a swollen bug bite on her R ankle. She admits about 4 days ago she was hiking near Lake Tahoe and tried to pet some of the wildlife, mainly squirrels and rabbits. Physical exam was unremarkable apart from the stated bug bite near the patient's ankle with some lymphadenopathy. The attending physician suspects the patient has Yersinia pestis from a flea bite from one of the animals due to an increase in sylvatic plague in the area. A culture of the infected bite proved his suspicions. Which of the following is most likely the bacteria's virulence factor? A. The bacteria's ability to utilize surface adhesion proteins on endothelial cells, attracting macrophages, escaping degradation. B. The bacteria's capsule, it is antiphagocytic and is resistant to serum killing. C. The bacteria's ability to produce pili and adhere to endothelial cells. D. The bacteria's ability to produce toxins LT1 & STa. E. The bacteria's ability to invade M cells and avoid the phagosome and degradation.

A. Incorrect: This pertains to Rickettsia Rickettsii - Rocky Mountain Spotted Fever; it has similar symptoms as stated in the question stem, but it is not pertaining to Enterobacteriaceae. B. Correct: Yersinia has multiple virulence factors, but Yersinia pestis is resistant to serum killing and is antiphagocytic. C. Incorrect: E. coli uses pili for adherence in regard to urinary tract infections; it is the leading cause of UTIs. Though Yersinia pestis does adhere to surfaces, it does not have pili. D. Incorrect: the toxins are associated with ETEC - they are involved with Traveler's Diarrhea. E. Incorrect: Shigella and Salmonella invade M cells, causing damage to the epithelial cells.

A 3-year-old female presents to the ER with initially watery diarrhea that has become bloody, abdominal pain, and signs of severe dehydration. The patient's parents state that symptoms started about 3 days after they gave the child fruit juice from a local farm. If the family refuses treatment, what condition still remains a risk for the patient? A. Neonatal Meningitis B. UTI C. Hemolytic Uremic Syndrome D. Traveler's Diarrhea E. Dysentery

A. Neonatal Meningitis -Incorrect because it is a CNS infection, will present with fever and other associated CNS symptoms. B. UTI - Incorrect because a UTI does not present this way and would not be common in a 3-year-old female. C. Hemolytic Uremic Syndrome - Correct because STEC (EHEC) produces Shiga-like toxin (Stx2) that is associated with breakdown of the glomerular endothelium. D. Traveler's Diarrhea - Incorrect because this is caused by ETEC and does not present with bloody diarrhea. E. Dysentery - Incorrect because it presents with a fever and is very rare.

An 8-year-old girl is taken to her pediatrician because she is experiencing fluid loss, frequent non bloody diarrhea, and a fever. She had started experiencing these symptoms after receiving brand new livestock and animal feed at her family's farm. The provider knows that the patient is dealing with Salmonella, bacteria that invade through M Cells, but she does not want it to lead to a bacteremia. Which of the following is the correct serotype for this patient, so the provider can treat her correctly? A. Salmonella enterica, serovar. Typhi B. Salmonella enterica, serovar. Infantis C. Salmonella enterica, serovar. Enteritidis D. Salmonella enterica, serovar. Choleraesuis E. Salmonella enterica, serovar. Paratyphi

A. Salmonella Enterica, serovar. Typhi - Wrong because this serovar is not adapted to animals and humans. This is only adapted to humans and is related to enteric fever. B. Salmonella Enterica, serovar. Infantis - Wrong because this serovar is not adapted to animals and humans. C. Salmonella Enterica, serovar. Enteritidis - Wrong because this serovar is not adapted to animals and humans. D. Salmonella Enterica, serovar. Choleraesuis - Correct answer because this is the only one from the answer choices that is adapted to animals and can infect humans. E. Salmonella Enterica, serovar. Paratyphi - Wrong because this serovar is not adapted to animals and humans. This is only adapted to humans and is related to enteric fever.

A patient with a prior cardiac history of a prosthetic valve placement comes to the emergency department for weakness and shortness of breath. The emergency department physician orders a CBC test to determine the patient's blood count. The patient is diagnosed with anemia and microscopic inspection of their blood is performed. What is the most likely cause of the patient's anemia and what shape would be present in the patient's RBCs? A. The anemia is most likely caused by a bioprosthetic valve and the RBCs would look normal B. The anemia is most likely caused by a mechanical valve and the RBCs would have schistocytes C. The anemia is most likely caused by a bioprosthetic valve and the RBCs would have schistocytes D. The anemia is most likely caused by hemolytic uremic syndrome (HUS) and would have normal RBCs E. There is not enough information in the question to determine how the patient could be anemic

A. The anemia is most likely caused by a bioprosthetic valve and the RBCs would look normal Incorrect. Although traumatic anemia can be caused by a bioprosthetic valve, a mechanical valve is a lot more likely to cause traumatic anemia. Plus the RBCs would not look normal and would have schistocytes. B. The anemia is most likely caused by a mechanical valve and the RBCs would have schistocytes Correct. Mechanical valve replacements are much more likely to cause traumatic anemia than bioprosthetic bovine or porcine valves. The RBC examination would also have schistocytes so this is the correct answer. C. The anemia is most likely caused by a bioprosthetic valve and the RBCs would have schistocytes Incorrect. As mentioned above, traumatic anemia can be caused by a bioprosthetic valve but mechanical is much more likely. Both are right but mechanical is the more correct choice D. The anemia is most likely caused by hemolytic uremic syndrome (HUS) and would have normal RBCs Incorrect. HUS is implicated in traumatic anemia but would also have schistocytes instead of a normal RBC smear. E. There is not enough information in the question to determine how the patient could be anemic · Incorrect. The patient history shows a prior prosthetic valve placement should give an idea that either a mechanical or bioprosthetic valve might be involved in the anemia.

A 19-year-old patient presents to the ED with cramps, vomiting, nausea, a low-grade fever, and watery, non-bloody diarrhea two days after returning home from a trip to Belize. PCR analysis of the fecal specimen suggests that the patient's diarrhea is due to inhibition of fluid absorption and an increase in chloride secretion in the small intestinal epithelium due to infection with enterotoxigenic E. coli (ETEC). The patient's diarrhea is caused by which of the following? A. STa enterotoxin-mediated increase in cGMP as well as LT-I mediated increase in cAMP. B. STb enterotoxin-mediated increase in cGMP as well as LT-II mediated increase in cAMP. C. STa enterotoxin-mediated increase in cAMP as well as LT-I mediated increase in cGMP. D. STb enterotoxin-mediated increase in cGMP as well as LT-II mediated increase in cGMP. E. LT-I enterotoxin-mediated increase in cAMP.

A. This is the correct answer and can be found on page 257. STa and LT-1 enterotoxins are most commonly associated with human infection of ETEC. STa mediates an increase in cGMP and LT-1 mediates an increase in cAMP. B. Incorrect. STb enterotoxin is not usually associated with human infections of ETEC. C. Incorrect. STa enterotoxin mediates an increase in cGMP and LT-I mediates an increase in cAMP. D. Incorrect. STb and LT-II enterotoxins are not usually associated with human infections of ETEC. E. Incorrect. Although LT-I enterotoxin does mediate an increase in cAMP, this is not the only enterotoxin at play in this particular strain of ETEC, because the question stem states that this patient's diarrhea is resulting from two different events. An increase in cAMP causes the hypersecretion of chloride, thus reducing its absorption, but the increase in cGMP mediated by STa inhibits fluid absorption and is not specific to chloride absorption.

A child of Mediterranean descent presents to the clinic with fatigue, facial bone deformities, slowed development and hepatosplenomegaly. After examining lab results along with observed signs and symptoms, the physician diagnosed the child with a type of hereditary anemia. What is the general pathogenesis of this blood disorder? A. Autosomal dominant disorder caused by mutations that affect the red cell membrane skeleton B. Autosomal recessive disorder resulting from a mutation in β-globin that causes deoxygenated hemoglobin to self-associate into long polymers C. Autosomal codominant disorders caused by mutations in α- or β-globin that reduce hemoglobin synthesis D. X-linked disorder caused by mutations that destabilize G6PD, making red cells susceptible to oxidant damage E. Disorder caused by antibodies against normal red cell

A. incorrect: this is true of Hereditary Spherocytosis B. incorrect: this is true of Sickle Cell anemia C. Correct: this is true of Thalassemias in general. The symptoms mentioned in the question stem are specific to a type of Thalassemias disorder called Beta-Thalassemia major. D. Incorrect: This is true of Glucose-6-Phosphate Dehydrogenase Deficiency E. Incorrect: This is true of Immunohemolytic Anemias

Patient comes to the hospital with a fever, headache, and redness. When the physician looked at the patient white blood cell differentials, he noticed that the patient had an early, acute inflammatory reaction when he noticed elevations of which leukocyte and its granules contained lysosomal hydrolase and myeloperoxidase ? A) polymorphonuclear leukocyte B) Basophil C) monocyte D) eosinophil E) macrophage

Ans : A) polymorphonuclear leukocyte aka Neutrophil; they are the main phagocyte in the early inflammatory site, first wave to arrive at tissue damage. Also the granules of segmented neutrophil contain lysosomal hydrolase and myeloperoxidase A) Correct answer-- see above B) basophil is wrong since its granules contain histamine, heparin, and myeloperoxidase C) is wrong because it is used for later stage of the inflammatory response D) is wrong since it is for later phase of inflammation E) precursor of macrophages; so it is also used in later stage of the inflammatory response

A 4 year old boy presents to the ER with abdominal pain and blood in his stools that started yesterday. Mom stated that since it is summer they went to a farm where the child drank fresh milk from a cow. The abdominal pain started 3 days ago and has been getting worse along with some vomiting but no fever. Lab tests showed that the organism that grows colorless on Sorbitol-MacConkey agar. The doctor is worried about complications from this organism. What would be the best treatment for this child? A. Vaccination B. Broad spectrum antibiotic regimen C. Supportive treatment D. Narrow spectrum antibiotic regimen E. Fecal transplant

Answer Explanation: C is correct because: Supportive treatment is used to provide symptomatic relief in patients who are suffering from STEC. The physician is worried about increasing the risk for the child getting HUS. Any use of antibiotics could increase the risk of secondary complications of STEC. A. Incorrect - Vaccination will not work on STEC. Rather, it can be useful when dealing with Salmonella typhi B. Incorrect - Any use of antibiotics may worsen the infection by causing the bacteria to become stressed thereby causing the release of more toxins C. Correct - see above D. Incorrect - Any use of antibiotics may worsen the infection by causing the bacteria to become stressed thereby causing the release of more toxins E. Incorrect - Fecal transplants are only used to correct for normal colon flora. STEC is not an opportunistic pathogen like C. difficile so a fecal transplant will not work.

A 26-year-old male was exploring an old sewer when he discovered a pack of rats feeding on some garbage that had been left behind. He decided to pick up one of the rats but was unsuccessful as they ran away. As he was walking out of the sewer, he noticed that he had gotten a flea bite while in there. He did not think anything of it and continued on with his day. 5 days later, he went to the hospital with a high fever and painful bubo in his groin area. The patient was not treated quickly enough and developed bacteremia and later died. What is the pathogen that killed this patient? A. Yersinia enterocolitica B. Yersinia pestis C. Klebsiella granulomatis D. Salmonella typhi E. Yersinia pseudotuberculosis

Based off of the patient's symptoms, he developed the Bubonic plague which is carried by rats and fleas after biting the infected rat. The symptoms include: high fever, bubo either in the groin or axilla area and bacteremia if the patient is untreated or not treated fast enough. Yersinia pestis is the only pathogen that causes the Bubonic plague. A. Incorrect - is a common cause of enterocolitis and is associated with ingestion of contaminated food or water. B. Correct answer - see above. C. Incorrect - although it is a disease that can affect the groin area, it is transmitted through sexual intercourse or nonsexual trauma and is not associated with a high fever. D. Incorrect - causes a fever but is not is highly adapted to humans and does not cause disease in nonhuman hosts. E.Incorrect - is a relatively uncommon cause of human diseases

A hematologist is preparing a presentation for undergraduate students regarding different cells that are found in the blood that pertain to her research involving evasion of the host immune response. After informing students about the functions of lymphocytes, basophils, eosinophils, which of the following statements is incorrect? A)Lymphocytes are the most common agranulocytes, are not terminally differentiated and have three functionally distinct types: T lymphocytes (involved in cell mediated immunity), B lymphocytes (involved in the production of circulating antibodies) and NK cells (kill virus infected and some types of tumor cells). B) Basophils account for less than 0.5% of total leukocytes, contain Fc receptors for IgE antibodies and CD40L proteins on the plasma membrane surface. The cytoplasm of these cells can have primary granules (variety of lysosomal acid hydrolases) or secondary granules containing heparin, histamine, heparan sulfate, leukotrienes, IL-4 and IL-13. C) Neutrophils are the most common granulocytes and are motile cells and are able to migrate to the area of tissue damage by the expression of adhesion molecules on the neutrophil surface. D) Eosinophils are bilobed cells associated with allergic reactions, parasitic infections and chronic inflammation. Eosinophils have primary granules that contain lysosomal acid hydrolases and secondary granules that contain a crystalloid body made of major basic protein (MBP), eosinophil cationic protein (ECP), eosinophil peroxidase (EPO), and eosinophil-derived neurotoxin (EDN) that can be seen with the TEM. E) Basophils are functionally related to and identical to mast cells of the connective tissue. Both basophils and mast cells originate from basophil-mast cell progenitor (BMCP) cell. Both basophils and mast cells bind to IgE (antibody secreted by plasma cells) binds to Fc receptors after allergen/antigen exposure that triggers the activation of these basophils and mast cells that will go on to release vasoactive agents that are associated with the vascular disturbances associated with hypersensitivity and anaphylaxis.

Choices A, B, C and D are all true statements. Choice E is the correct answer. Although basophils are functionally related to, they are NOT identical to mast cells. The rest of the information is correct.

A 43-year-old female presents to her local clinic with fatigue, shortness of breath, pale skin, and abdominal pain. The patient notifies her attending physician that she has had episodes of bright red blood appearing in her urine, at night. The physician orders a flow cytometry test, which reveals an abundance of red blood cells that are deficient in glycosylphosphatidylinositol-linked membrane proteins. Which of the following best explains the reason for the lack of such proteins? A. Random inactivation of the X-linked PIGA gene B. Deletion of two 𝛂-globin genes C. Antibodies that recognize and lead to RBC destruction D. Cytoskeleton defects E. Abnormalities in glutathione metabolism

Correct Answer - A A. Random inactivation of the X-linked PIGA gene · Correct.The symptoms are characteristic of Paroxysmal Nocturnal Hemoglobinuria (PNH), a type of hemolysis that results from an acquired mutation. The disease can be characterized by lyonization (random inactivation of an X chromosome), which in turn affects the X-linked phosphatidylinositol glycan complementation group A (PIGA) gene. This gene is essential for the synthesis of certain membrane-associated complement regulatory proteins, particularly glycosylphosphatidylinositol (GPI) - membrane proteins. The lack of these proteins may lead to RBC hemolysis, and periodic increases in hemoglobin that are detectable in urine. The presence of red blood cells that lack the GPI - linked proteins can be detected using flow cytometry, the go to laboratory technique to diagnose PNH. B. Deletion of two 𝛂-globin genes · Incorrect. The patient is suffering from PNH, as per the symptoms and diagnosis. This answer choice indicates a diagnosis of ⍺-thalassemia, which is characterized by small red blood cells (microcytosis) and minimal anemia C. Antibodies that recognize and lead to RBC destruction Incorrect. PNH is not an autoimmune disorder. PNH is characterized by the lyonization of an X-chromosome which inactivates the PIGA gene. This answer choice is indicative of immunohemolytic anemia D. Cytoskeleton defects Incorrect. Cytoskeleton defects are characteristic of hereditary spherocytosis, where red blood cells go from a biconcave, "donut"-shape to a spherical shape. This can manifest in splenomegaly, which is not observed in the patient. E. Abnormalitiesin glutathione metabolism Incorrect. Abnormalities in glutathione metabolism are characteristic of glucose-6-phosphate dehydrogenase deficiencies, which affect the pentose-phosphate pathway, not PNH. This eventually leads to a lack of glutathione, leaving RBCs more prone to oxidative stress. These symptoms are not characteristic of glucose-6-phosphate dehydrogenase deficiency. Symptoms of this deficiency often manifest when an individual is exposed to oxidative stressors, such as fava beans or antimalarial drugs.

A 25-year-old male patient presents to the clinic with complaints of fatigue, shortness of breath, and pale skin. Previous medical history reveal that the patient was diagnosed with Alpha Thalassemia. Further lab diagnostics were taken and indicated that the patient has a genetic deletion of two alphaglobin genes from chromosome 16. Based on the deletion of the two alpha-globin genes, what other lab results will be seen in the patient? A. Decreased concentration of Beta-globin chains B. Microcytic Red blood cells and reduced hemoglobin concentration C. Signs for megaloblastic anemia D. Precipitated erythroid precursors due to excess alpha-globin chains E. Deficiency in folate or vitamin B12

Correct Answer - B: Alpha Thalassemia indicates a low concentration of alpha-globin chains. Losing two alpha-globin genes results in small red blood cells and low concentrations of hemoglobin. This is because the concentration of alpha-globin chains become necessary for cell size and hemoglobin formation. For hemoglobin to be synthesized, there needs to be an even ratio of alpha-globin chains and beta-globin chains in a 1:1 ratio. Low amounts of alpha-globin chains combined with two missing alpha-globin genes strongly indicates these lab results A. Incorrect- Alpha Thalassemia does not indicate decreased beta-globin chain concentration B. Correct- See above C. Incorrect- Megaloblastic anemia indicates larger than normal red blood cells D. Incorrect- There is a decrease in alpha-globin chains, not an increase E. Incorrect- This represents characteristics for nutritional anemia, usually having larger than normal red blood cells

A 34 year old male presented to his local emergency department for a severe bacterial infection of his upper respiratory tract. To combat the infection, the attending physician started the man on a 14 day intravenous cephalosporin treatment. However, after 10 days of receiving this treatment, he began developing symptoms not typical for pharyngitis. The patient displayed signs of fatigue, severe weakness, shortness of breath and pale, cold skin to the touch. A Direct Coombs Antiglobulin test was then performed which ultimately resulted in significant clumping of the blood. Based on the patient's presentation and the diagnostic test performed, what is the most likely mechanism of action causing the patient's symptoms? A. Intravascular hemolysis due to IgE antibodies binding to the host's red cells at 2°C B. Interrupted synthesis of CD59 regulatory protein due to a mutation in PIGA C. Extravascular hemolysis due to IgG antibodies binding to the host's red cells at 37°C D. Direct hemolysis due to the destruction of the RBC membranes from Heinz Bodies E. A mutation in band 4.2 on the surface of the patient's RBCs

Correct Answer - C A. Intravascular hemolysis due to IgE antibodies binding to the host's red cells at 2°C Incorrect. The pt's symptoms are due to antibody binding against his RBCs, but not in these conditions. Immunohemolytic Anemia does not result in intravascular hemolysis and at 2°C, IgM would be the primary antibody of action, not IgE. B. Interrupted synthesis of CD59 regulatory protein due to a mutation in PIG. Incorrect. The patient is suffering from Immunohemolytic Anemia. This answer describes the mechanism of Paroxysmal Nocturnal Hemoglobinuria (PNH) C. Extravascular hemolysis due to IgG antibodies binding to the host's red cells at 37°C Correct. The pt is suffering from Immunohemolytic Anemia due to antibody production against the pt's RBCs at these conditions. D. Direct hemolysis due to the destruction of the RBC membranes from Heinz Bodies Incorrect. The patient is suffering from Immunohemolytic Anemia. This answer describes the mechanism of G6P-D Deficiency Hemolytic Anemia E. A mutation in band 4.2 on the surface of the patient's RBCs Incorrect. The patient is suffering from Immunohemolytic Anemia. This answer describes the mechanism of Hereditary Spherocytosis.

A 28-year-old male presents to the emergency department after suffering impalement to the abdomen during a car crash. The patient suffered from massive blood loss and presented to the primary care physician with symptoms of pale skin, weakness, and decreased blood pressure. The primary care physician orders a CBC and Differential and from further questioning and lab results during the patients week long recovery in the hospital, the primary care physician diagnoses the patient with Acute Blood Loss Anemia due to the trauma the patient encountered. What results from the CBC and Differential led to the primary care physicians diagnosis? A. Decrease in platelet production and white blood cell activity B. No effect in platelet production or white cell activity C. Decreased hematocrit and decreased reticulocytes D. Decreased hematocrit and increased reticulocytes E.Decreased hematocrit and increased hemoglobin concentration

Correct Answer - D A. Decrease in platelet production and white blood cell activity Incorrect. Acute Blood Loss Anemia will affect both platelet production and white blood cells, in addition to affecting red blood cell levels. The massive loss of blood will lead to a decrease in blood pressure, resulting in release of adrenergic hormones that mobilize granulocytes, promoting leukocytosis, which is an increase in white cell production not a decrease. Early recovery from blood loss can result in thrombocytosis, which is an increase in platelet production not a decrease. B. No effect in platelet production or white cell activity Incorrect. Acute Blood Loss Anemia does not only affect red blood cells. It also does have an effect on white cell activity and platelet production as well. Acute Blood Loss Anemia will mobilize granulocytes, therefore promoting leukocytosis and early recovery from blood loss will result in an increase in platelet production. C. Decreased hematocrit and decreased reticulocytes Incorrect. Acute Blood Loss Anemia will result in a decreased hematocrit, due to the massive hemorrhage and the body compensating for the loss in red blood cells via fluid movement into the intravascular component. However, the trauma will cause an increased number of reticulocytes, not decreased, due to the bone marrow producing more red blood cell precursors to compensate for the loss in red blood cells in the hemorrhage. D. Decreased hematocrit and increased reticulocytes Correct. In Acute Blood Loss Anemia due to trauma, the massive loss of blood volume will be restored by movement of fluid from the interstitial fluid to the intravascular component resulting in hemodilution and therefore a decrease in hematocrit. This decrease in hematocrit will cause tissue hypoxia, resulting in increased secretion of erythropoietin from the kidney which causes an increase in the number of reticulocytes. E. Decreased hematocrit and increased hemoglobin concentration· Incorrect. Acute Blood Loss Anemia will result in a decreased hematocrit, due to the massive hemorrhage and the body compensating for the loss in red blood cells via fluid movement into the intravascular component. However, there will be a decrease, not increase, in hemoglobin concentration because the body's oxygen supply is drastically reduced to the decreased number of oxygen-carrying red blood cells.

Researchers in a lab were comparing blood smear samples from five different patients. After staining each sample with H&E to observe the erythrocytes, they noticed one of the patients blood smear possessed abnormally shaped erythrocytes. Instead of a biconcave disc shape, they found spherical shaped erythrocytes. After further research and tests it was stated that the patient had hereditary spherocytosis. Why would a patient with hereditary spherocytosis be at risk for premature destruction of erythrocytes or hemolysis? A. Spherocytosis results from an autosomal recessive disorder that causes spontaneous hemolysis of the erythrocytes. B. Spherocytosis occurs from an autosomal dominant disorder that causes an extra lipid layer to form around the erythrocyte leading to a spherical shape which triggers hemolysis. C. Spherocytosis increases flexibility of the cytoskeleton of the erythrocyte causing the cell to burst from the increased membrane protein interactions. D. Spherocytosis occurs from defects in anchoring proteins causing the erythrocytes plasma membrane to detach and peel off, this leads to the inability of the erythrocyte to adapt to changes in the environment, ultimately resulting in hemolysis. E. Spherocytosis is a disorder where an increase in erythrocyte integral membrane proteins used for anchoring the plasma membrane to the cytoplasm causes the spherical shape of the erythrocyte, additionally the increase in interactions causes the cell to hemolyse in high pressure environments.

Correct Answer : D Spherocytosis is an autosomal dominant disorder that affects the ankyrin complex proteins, causing the erythrocytes plasma membrane to detach from the cytoplasm resulting in the round shape. The shape is not as flexible or stable as a biconcave erythrocyte so the spherical erythrocyte is unable to adapt to changes in the environment (ex. Osmotic pressure) resulting in premature destruction or hemolysis. A - Incorrect : Hereditary spherocytosis is an autosomal dominant genetic disorder affecting the ankyrin complex proteins that function in anchoring the plasma membrane to the cytoplasm causing a spherical shape which is not as elastic or stable causing premature destruction / hemolysis. B - Incorrect : Spherocytosis causes a defect in the anchoring plasma membrane proteins that gives the erythrocyte a spherical shape because the plasma membrane detaches from the cytoplasm, there is no extra lipid layer. The spherical shape leads to premature destruction / hemolysis. C - Incorrect : Spherocytosis causes a defect in the ankyrin protein complex causing the plasma membrane to detach from the cytoplasm resulting in a spherical shape, the spherical shape is less flexible than a biconcave erythrocyte so it cannot adapt to changes in the environment which leads to hemolysis. D. - Correct : See above E - Incorrect : A defect in ankyrin complex proteins that function to anchor the plasma membrane to the cytoplasm causes the plasma membrane to peel off of the cytoplasm giving a spherical shape. The spherical shape does not have the ability to adapt to environment changes which leads to hemolysis / premature destruction.

A researcher in a clinical lab is investigating the potential of the CRISPR system for possible gene therapeutics. In efforts to test the effectiveness of CRISPER/Ca9, the researcher decides to knockout the gene responsible for the transaldolase enzyme in the Hexose Monophosphate Shunt and monitor levels of Fructose 6-phosphate production. To the researchers' surprise, Fructose 6-phosphate was still produced. What enzyme in the Pentose Phosphate Pathway may be responsible for the production of Fructose 6-phosphate? A. Gluconolactonase B. Hexokinase C. Glucose 6-phosphate dehydrogenase D. Epimerase E. Transketolase F. Glucokinase

Correct Answer : E - Transketolase in the Pentose Phosphate Pathway is responsible for two reactions. The first is to convert Xylulose 5-phosphate and Ribose 5-phosphate into Sedoheptulose 7-phosphate. However, Transketolase can also take a two-carbon keto fragment from Xylulose 5-phosphate and move it to Erythrose 4-phosphate forming Fructose 6-phosphate. A. Incorrect - Gluconolactonase is responsible for converting 6-Phosphoglucono-δ-lactone to 6-Phosphogluconate in oxidative phase of HMP shunt B. Incorrect - Hexokinase is responsible for phosphorylating Glucose to Glucose 6-phosphate C. Incorrect - Glucose 6-phosphate dehydrogenase is responsible for oxidizing Glucose 6-phosphate and reducing NADP+ to NADPH D. Incorrect - Epimerase is responsible for converting Ribulose 5-phosphate to Xylulose 5-phosphate E. Correct: see above explanation F. Incorrect - Glucokinase is responsible for phosphorylating Glucose to Glucose 6-phosphate in liver and pancreas

A 27-year-old male from Senegal presents to the clinic with complaints of occasional fatigue but no other symptoms. Lab results show increased MCHC with intracellular crystals seen on blood smear. Molecular analysis reveals the patient is homozygous for HbC mutation. What are the effects of this hemoglobin variant? A. It promotes water loss from the cell by activating K+ transporter and lowers the hemoglobin solubility by replacing glutamate to lysine, resulting in the precipitation of the hemoglobin variant. B. It inhibits water loss from the cell by activating K+ transporter and lowers the hemoglobin solubility by replacing glutamate to lysine, resulting in the precipitation of the hemoglobin variant. C. It inhibits water loss from the cell by activating K+ transporter and increases the hemoglobin solubility by replacing glutamate to lysine, resulting in the precipitation of the hemoglobin variant. D. It promotes water loss from the cell by activating K+ transporter and increases the hemoglobin solubility by replacing glutamate to lysine, resulting in the precipitation of the hemoglobin variant. E. It promotes water loss from the cell by activating K+ transporter and lowers the hemoglobin solubility by replacing glutamate to glycine, resulting in the precipitation of the hemoglobin variant.

Correct Answer A- Hemoglobin variant, HbC, results from a Glu-to-Lys replacement in the same position as the HbS mutation. This mutation promotes water loss from the cell by activating the K+ transporter, resulting in an abnormal concentration of hemoglobin within the cell. In addition, the replacement substantially lowers the hemoglobin solubility in the homozygote, resulting in a tendency of the mutant hemoglobin to precipitate within the red blood cell. A symptom of homozygotes for the HbC mutation is a mild hemolytic anemia, which explains the occasional fatigue the patient has been experiencing. A.Correct - See above B. Incorrect - It promotes water loss from the cell by activating K+ transporter C. Incorrect -It does not inhibit but rather promotes water loss from the cell by activating K+ transporter and lowers the hemoglobin solubility by replacing glutamate to lysine. D. Incorrect -It lowers the hemoglobin solubility by replacing glutamate to lysine, resulting in the precipitation of the hemoglobin variant. E. Incorrect -The amino acid replacement is from glutamate to lysine.

A researcher is conducting an experiment on hemopoiesis and has acquired several types of progenitor cells to experiment on. If the researcher wants to begin his work with NK cells, which of the following progenitor cells should he select? Megakaryocyte-committed progenitor cells Neutrophil progenitors Common lymphoid progenitor cells Monocyte progenitors Eosinophil progenitors

Correct Answer C- NK or natural killer cells are a type of lymphocyte, common lymphoid progenitor cells give rise to the various lymphocytes including T, B, and NK cells Incorrect- these cells give rise to erythrocytes and platelets. Incorrect- these cells give rise to neutrophils Correct see above Incorrect- these cells give rise to monocytes Incorrect- these cells give rise to eosinophils

A 1 y.o. male child presents to his PCP with general weakness, fatigue, loss of appetite, and irritability. Parents have been supplementing diet with PediaSure mix. Normal diet is well balanced. Upon testing, his red blood cells appear microcytic and hypochromic. Further questioning reveals that he has been teething and currently uses a newly thrifted 80 year old antique crib. What deficiency is causing the patient's symptoms? A.Vitamin B6 B. Folate C. Heme D. B12 E. G6PD

Correct Answer C- Older cribs may have paint that is lead-based. Babies can get lead poisoning from chewing and gnawing on a crib with lead-based paint.δ-ALA dehydratase, which contains zinc, and ferrochelatase are inactivated by lead. Thus, in lead poisoning, δ-ALA and protoporphyrin IX accumulate and the production of heme is decreased. Anemia results from a lack of hemoglobin, and energy production decreases because of the lack of cytochromes for the electron-transport chain. A. Incorrect- Vitamin B6 deficiency would result in microcytic and hypochromic red cells.Symptoms are due to decrease rate of heme production because the first step in heme synthesis, ALA synthase, requires pyridoxal phosphate. Which should be supplemented through the patient's pediasure supplement. B. Incorrect- Vitamin B12 and folate deficiency present with macrocytic cells. C. Correct- See above D. Incorrect- Vitamin B12 and folate deficiency present with macrocytic cells. E. Incorrect- Hemolysis occurs because the formation of NADPH is insufficient for the oxidant stress.Hemolysis results in anemia, which is usually normocytic and normochromic.

A 25 year old graduate student researcher is interested in the development of Erythrocytes. He exposes common myeloid progenitor cells (CMP) to erythropoietin, IL-3, and IL-4. He then decides to block expression of transcription factor GATA-1. What is the first precursor cell in erythropoiesis that the researcher is preventing formation of? A. proerythroblast B. basophilic erythroblast C. myeloblasts D. megakaryoblast E. basophil

Correct Answer Choice A. The proerythroblast is the first microscopically recognizable cell in erythropoiesis (formation of red blood cells). Expression of the transcription factor GATA-1 is required for CMP cells to turn into MEP cells, which eventually develop into proerythroblasts. A-correct answer (see above) B-incorrect, while basophilic erythroblast is part of Erythropoiesis it the 2nd precursor cell to develop after proerythroblasts. C-incorrect, myeloblast is the first recognizable cell that begins the process of granulopoiesis. D- incorrect, this cell type is part of the development of thrombocytes (Thrombopoiesis) E-incorrect, this is mature Granular leukocyte.

John Doe presents to the emergency department with complaints of abdominal pain, dysentery, lymphadenopathy, and a temperature of 100.1° F. Last week he was discharged from the hospital after having successfully completed invasive heart surgery during which he received a perioperative blood transfusion. After a physical exam, the physician notes acute lower right quadrant abdominal pain with point tenderness. A stool culture is ordered to confirm a diagnosis of Y. enterocolitica. Which of the following virulence factors is responsible for Yersinia spp. ability to secrete proteins into phagocytic cells that dephosphorylate proteins required for phagocytosis? A) YopJ/P gene product B) YpkA/YopO gene product C) YopE gene product D) YopH gene product E) YopD gene product

Correct Answer Choice D. Upon bacteria cell contact with phagocytotic cell, YopH gene product releases proteins with phosphatase activity resulting in dephosphorylation of proteins necessary for phagocytosis. A) YopJ/P gene product - Incorrect: YopJ/P responsible for initiation of apoptosis in macrophages B) YpkA/YopO gene product - Incorrect: YpkA/YopO is an effector protein termed Yersinia protein kinase A. Thus, has a phosphorylation function which is wrong. Also wasn't discussed in the text. C) YopE gene product - Incorrect: YopE induces cytotoxicity by disrupting actin filaments. D) YopH gene product - Correct. E) YopD gene product - Incorrect: Helps form translocation apparatus to deliver effector proteins into the eukaryotic cell. Also wasn't discussed in the text.

A 27-year-old male presents to his primary care physician with a pale appearance and describes his symptoms as malaise, general weakness and becoming fatigued rather easily. After a few tests, it was determined that this patient has developed an immunohemolytic anemia. What test did the attending physician use to diagnose the patient's condition based on his desire to characterize the antigens target and temperature dependence? A. A direct coombs antiglobulin test where the patient's serum is tested for its agglutinating ability against commercial red blood cells with specific antigens B. A direct coombs antiglobulin test where the patients red cells were mixed with a serum containing specific human complement activators C. A western blot where the patient's serum was reacted with both a primary and secondary antibody and analyzed on a PDVF membrane D. An indirect coombs antiglobulin test where the patient's serum is tested for its agglutinating ability against commercial red blood cells with specific antigens E. An immunoprecipitation assay where the patient's serum was concentrated and a gene expression analysis was performed to find the autoantibody sequence on his red cells.

Correct Answer D The correct answer is D because an indirect coombs antiglobulin test is used to characterize an antigens target and temperature dependence. Also, the indirect coombs test is performed with the patients' serum being testing for agglutination against commercially available red cells with a defined antigen. A. Incorrect: the direct coombs test does not test a patient's serum against commercial red blood cells and also does not characterize the antigens target and temperature dependence. This question is simply the indirect coombs definition but with the wrong test name B. Incorrect: a direct coombs does not characterize the antigens target and temperature dependence. This is the correct mechanism for a direct coombs test but does not diagnose the anemia based on the physician's desire in the question stem. C. Incorrect: a western blot is used to detect the presence of a protein in a sample. Based on an immunohemolytic anemia, you would have to use the patient's autoantibody as a primary antibody for this experiment, as well as develop a secondary antibody to bind the patients autoantibody for visualization on the PDVF membrane. Based on that, this experiment is essentially impossible, irrelevant and does not diagnose immunohemolytic anemia. D. Correct: See above for explanation E. Incorrect: In terms of lab techniques, 1. this is not how an immunoprecipitation assay is performed, and 2. it would not be relevant to diagnose the patients immunohemolytic anemia.

A patient comes to the ER with complaints of pelvic/abdominal pain, odor, and blood when urinating. She explains that she might have a recurring UTI from which she has been suffering from on and off for the past 2 years. After physical examination, the physician decides to obtain a bodily fluid swab. After viewing the swabs contents microscopically, it was confirmed that she had a UTI infection based on the increased levels of a certain cell type that initiates respiratory bursts which create oxygen radicals that rapidly destroy foreign particles. What physical characteristics might have the physician seen under a microscope to confirm his findings? A. Red granules, polymorphonuclear B. Blue granules, polymorphonuclear C. Agranular, blue staining, mono nuclear D. Pink granules, polymorphonuclear E. No nucleus present, granular, cytoplasmic organelles

Correct Answer D - Neutrophils migrate to areas of infection or tissue damage. They are phagocytic cells that engulf foriegn bodies and destroy them by initiating respiratory bursts that create oxygen radicals that rapidly destroy foriegn material at the site of infection. Neutrophils' main physical characteristics under a microscope, is that they are granular, polymorphonuclear leukocyte that stain pink. A. Incorrect; this describes eosinophils which are mainly seen for parasitic infections, asthma, allergic reactions, autoimmune diseases and some cancers and contain hydrolytic enzymes and cationic proteins. B. Incorrect; this describes basophils which are seen in allergic reactions and release histamine, B- glucosidases and lysophospholipases. C. Incorrect; this describes a lymphocyte which mainly secrete antibodies in response to antigen binding, may target virally infected and malignant cells for destruction and wouldn't be seen as predominantly as neutrophils D. Correct ; see above E. Incorrect; this describes thrombocytes which mainly function in blood clotting.

A 14 year girl comes into the ER with complaints of fatigue and shortness of breath. During the patient's physical examination, the physician finds a rash spread across the patient's back. Upon further testing, it is discovered that the patient has atopic dermatitis, causing extensive tissue damage. What would be the action of the first type of leukocyte to arrive at the site of infection? A. Release of histamine to increase vascular permeability B. Secrete antibodies in response to the antigen-binding occurring at the site of infection C. Activate lysosomes to increase the activity of hydrolytic enzymes and cationic proteins D. Initiate a respiratory burst which creates oxygen radicals to destroy the foreign material E. Engulf microorganisms and necrotic host cell debris

Correct Answer D: The first immune cell to arrive at the site of any infefction is a neutrophil. Neutrophils are granulocytes that migrate rapidly to areas of infection or tissue damage to engulf any foreign bodies present. After engulfment, neutrophils create respiratory bursts which generate the production of oxygen radicals to destroy the foreign material they ingested. A. INCORRECT- This is not a function of neutrophils. This a function of basophils, which are granulocytes that participate in hypersensitive reactions. B. INCORRECT- This is not a function of neutrophils. This is a function of B-cells which are derived from mononuclear leukocytes known as lymphocytes. C. INCORRECT- This is not a function of neutrophils. This is a function of eosinophils, which are granulocytes that protect against parasites and remove fibrin during inflammation. D. CORRECT- See above E. INCORRECT- This is not a function of neutrophils. This is a function of macrophages, which are derived from mononuclear leukocytes known as monocytes. Macrophages tend to arrive after a granulocyte attacks foreign material.

A fifteen-year-old female is brought into the emergency room with symptoms of vomiting, abdominal pain and bloody diarrhea, but is afebrile. She mentions that she recently consumed tacos containing ground beef a few days prior but notes that the ground beef may have been slightly undercooked. Which of the following is most likely the pathogenesis of the organism that is causing her symptoms? A. Shiga Toxins (Stx1 and Stx2) that cleave 28s rRNA to disrupt protein synthesis B. Heat-stable ST and heat-labile LT toxins that stimulate hypersecretion of fluids by respectively increasing cGMP and cAMP C. Destruction of the microvillus with attachment/effacement histopathology D. Bacterial invasion and destruction of colonic epithelium E. Autoagglutination in a "stacked-brick" arrangement over small intestine epithelium

Correct Answer is A: The patient's symptoms of abdominal pain, vomiting, and bloody diarrhea are caused by Shiga toxins (Stx1 and Stx2) of STEC. STEC is associated with consumption of undercooked beef. A: see above B: This is incorrect because ST and LT toxins are involved in ETEC which is associated with watery diarrhea. C: This is incorrect because this pathogenesis is involved with EPEC which leads to watery diarrhea. D: This is incorrect because this is the pathogenesis of EIEC which involves watery diarrhea that progresses to bloody diarrhea with leukocytes. E: This is incorrect because this is the pathogenesis involved in EAEC that leads to watery diarrhea.

A 24-year-old male presents to the clinic with complaints of extreme fatigue, weakness and pale skin appearance. Lab evaluation including CBC, reticulocyte count, ferritin test and peripheral blood smear indicate a normal RBC count with abnormally low ferritin in the blood. The red blood cells also appear to be small and pale indicating microcytic and hypochromic anemia. What is the cause of the microcytic red blood cells? A. Patient has a folate or vitamin B12 deficiency B. Developing red blood cells continue to divide past their normal stopping point C. Patient has Epo receptor mutation D. Patient has a vitamin B6 deficiency E. Differentiating hematopoietic cells remain in an immature and proliferative state

Correct Answer is B. Iron deficiency will result in a microcytic, hypochromic anemia. RBCs will be small and pale. The lack of iron results in decreased heme synthesis, which in turn affects globin synthesis. Maturing red cells following their normal development program divide until their hemoglobin has reached the appropriate concentration. Iron-deficient developing RBCs continue dividing past their normal stopping point, resulting in small (microcytic) red cells. The cells are also pale because of the lack of hemoglobin, compared with normal cells, thus a pale microcytic anemia result. (P. 880 & 883) A) Incorrect - A blood smear of a patient that has a folate or vitamin B12 deficiency would show megaloblastic anemia, in which the cells are larger than normal (P.883) B) Correct - see above C) Incorrect - A patient with an Epo receptor mutation has a higher-than-normal percentage of RBCs in the circulation because the mutant Epo receptor cannot be deactivated by SHP-I (P.882) D) Incorrect - A patient with a vitamin B6 deficiency would have microcytic, hypochromic anemia and iron stores are usually elevated. The amount of ferritin in the blood is the most sensitive indicator of the amount of iron in the body's stores. The ferritin test showed abnormally low ferritin indicating that patient has low iron stores (P.876). E) Incorrect - Leukemias arise when a differentiating hematopoietic cell does not complete its development program but remains in an immature, proliferative state. The patient signs and symptoms indicate he has an iron deficiency anemia not leukemia (P.879)

A new breastfeeding mother presents to her OB/GYN concerned that she is not producing breastmilk because of her low lactose intake. Is this a reasonable concern as to why the new mother has a low production of breastmilk? A. Yes, because in order to produce lactose she must intake lactose from some dietary source. B. Yes, because in order to produce lactose the mother must convert lactose to Ca2+ which is the main component of breast milk. C. No, because lactose in the mammary gland is derived from the epimerization of glucose. D. No, because breastmilk is mainly synthesized from dietary Ca2+, not dietary sugars. E. No, it's not necessary but in order to produce maximal amounts of breastmilk you must and take lactose.

Correct Answer is C. As explained above, because lactose in the memory gland is derived principally from the epimerization of UDP-glucose. A. Incorrect, because ingestion of lactose is not needed for the production of breastmilk. B. Incorrect, because ingestion of lactose is not needed for the production of breastmilk. C. Correct, see above. D. Incorrect, because breastmilk is synthesized from glucose intake. E. Incorrect, because lactose in breastmilk is synthesized from glucose rather than lactose.

A 25-year-old female patient visits her primary care physician and complains of fatigue, lethargy, and pale yellow tinge on her skin. The doctor orders a CBC and blood smear. Her hemoglobin is 10g/dL (normal is 12-16g/dL). Her RBC count is 3.1x1012L (normal is 4.2-5.4x1012L). The blood smear shows larger than normal RBCs. What is the most likely diagnosis for this patient? A. Microcytic anemia B. Iron deficiency C. Vitamin B12 deficiency D. Sickle cell anemia E. Beta Thalassemia

Correct Answer is C: Vitamin B12 deficiency or Folate deficiency. Her RBC count has decreased, indicating anemia. Blood smear indicates megaloblastic condition. Vitamin B12 and folate are necessary for DNA synthesis. This nutrient deficiency causes megaloblastic anemia because the cell volume increases while the DNA synthesis decreases, leasing to larger volume but fewer RBCs made. A. Incorrect - The blood smear shows large blood cells which would not indicate microcytic anemia. B. Incorrect - Iron deficiency is a microcytic anemia condition & the RBCs are smaller and paler than normal due to decreased heme and globin synthesis. C. Correct. D. Incorrect - Sickle cell anemia is a genetic disease that causes sickling of the cells. The would be easily identifiable in a blood smear with their classic sickle shape. It is also classified as more of a microcytic anemia. E. Incorrect - Beta Thalassemia occurs due to a genetic mutation and is also considered microcytic anemia. If the person has both genes mutated, the RBCs size would decrease much more compared to if they're heterozygous for the mutation.

A 5 year old male patient was brought to the clinic because his parents are concerned about bloody diarrhea. The patient was previously healthy but started developing symptoms after attending a birthday party at a petting zoo. Lab test revealed that the microbe responsible for the GI infection is a gram-negative rod that belongs to the Enterobacteriaceae family. Which of the following lab results are expected for bacteria of this family? A. Glucose fermenter, Catalase +, oxidase - B. Glucose fermenter, Catalase -, oxidase + C. Glucose non-fermenter, Catalase +, oxidase + D. Glucose fermenter, Catalase -, oxidase - E. Glucose non-fermenter, Catalase +, oxidase -

Correct Answer: A - "The Enterobacteriaceae have simple nutritional requirements, ferment glucose, reduce nitrate, and are catalase positive and oxidase negative." This is the only correct answer with 3 right choices!

Susan was involved in an accident resulting in significant blood loss. Fortunately they were able to stabilize her but she required a blood transfusion to replace the large loss of blood volume. Susan was given O- blood because they did not know her blood type at the time. It was later determined Susan was A+, which of the options below indicates her blood type? A) N-Acetylgalactosamine and D antigen B) N-Acetylgalactosamine and Galactose C) N-Acetylgalactosamine and C antigen D) Galactose and C antigen E) Galactose and D antigen

Correct Answer: A - ABO and Rh blood typing are dependent on enzymatic synthesis processes on glycophorins present in the extracellular domains of blood cells. N-acetylgalactosamine is found in blood type A, where blood type B contains galactose at the top most component of the glycophorin. Additionally, AB blood types contain enzymes for both types, and O types contain neither, with its terminal end consisting of just galactose-fucose. In conjunction with ABO blood typing is Rh blood typing, which is indicated by the presence or lack of the D antigen, being positive and negative respectively. The various blood types then are determined by which of these are present or not on the blood cells A) N-Acetylgalactosamine and D antigen - CORRECT - This is the correct response for A+ blood type B) N-Acetylgalactosamine and Galactose - INCORRECT - this represents AB blood type without any info on the Rh antigen C) N-Acetylgalactosamine and C antigen - INCORRECT - this is the correct indicator for A blood type, but C antigen does not determine Rh status D) Galactose and C antigen - INCORRECT - this represents B type and C antigen does not determine Rh status E) Galactose and D antigen - INCORRECT - this represents B type and Rh+ blood

A 31-year old woman has just given birth to a child and is having trouble breastfeeding. When tests are performed, results show a lack of lactose and an excess of glycoproteins. Which enzyme catalyzes the last step of the defective pathway, and which subunit appears to be missing from the enzyme? a) Lactose synthase; α- lactalbumin b) Lactose synthase; galactosyltransferase c) Lactose dehydrogenase; α-lactalbumin d) Epimerase; galactosyltransferase e) Epimerase; α-lactalbumin

Correct Answer: A - The "defective pathway" in question is the lactose synthesis pathway. Lactose is formed in the mammary glands after childbirth when lactation is stimulated. Lactose synthase is the last enzyme in the lactose synthesis pathway, catalyzing the conversion from UDP-galactose to lactose. Lactose synthase has 2 protein subunits, α- lactalbumin and galactosyltransferase. In the absence of α- lactalbumin, galactosyl units are converted to glycoproteins instead of lactose. Therefore, the correct answer is A, with lactose synthase being the enzyme catalyzing the last step of the pathway and α- lactalbumin being the missing subunit. a) Correct: see above b) Incorrect- Lactose synthase is correct, but galactosyltransferase is the wrong subunit c) Incorrect- Lactose dehydrogenase is not in the lactose synthesis pathway; α- lactalbumin is the correct subunit d) Incorrect- Epimerase is in the lactose synthesis pathway, but it converts UDP-glucose to UDP-galactose, which is not the last step in the pathway; galactosyltransferase is the wrong subunit e) Incorrect- Epimerase is in the lactose synthesis pathway, but it converts UDP-glucose to UDP-galactose, which is not the last step in the pathway; α- lactalbumin is the correct subunit

A 39-year-old male of Mediterranean descent presents to his physician with complaints of fatigue, pallor, and shortness of breath. The patient says that he is currently taking aspirin for his heart condition and did not eat anything out of the ordinary except for some fava beans in a meal a few days ago. The physician orders a laboratory test to confirm his suspicion and diagnoses the patient with hemolytic anemia. Which enzyme is deficient in this patient and what can be seen on a blood smear because of this? A. Glucose-6-Phosphate Dehydrogenase, Heinz bodies B. Glucose-6-Phosphate Dehydrogenase, Negri bodies C. Pyruvate Dehydrogenase, Guarneri bodies D. Glycogen Phosphorylase, Heinz bodies E. Pyruvate Dehydrogenase, Negri Bodies

Correct Answer: A - The patient is diagnosed with hemolytic anemia based on his lab work and the symptoms he presented. Most G6PD mutations are found in patients with African or Mediterranean descent. Many drugs such as aspirin, sulfonamides, and antimalarials can trigger hemolysis. Furthermore, fava beans (favism) also develop hemolysis with the Mediterranean variant of G6PD. Heinz bodies indicate oxidative injury to erythrocytes and are a causative factor for hemolytic anemia. A. Correct- See Above B. Incorrect- Although Glucose-6-Phosphate is the correct enzyme, Negri bodies are an indication for the Rabies virus and can be found in neurons. C. Incorrect- Pyruvate Dehydrogenase deficiency would result in an accumulation of lactic acid, alanine, and pyruvate. The signs and symptoms are not present here. Guarneri bodies are cellular features in epithelial cells of those suspected to have poxvirus. D. Incorrect- Glycogen Phosphorylase is present in the muscles, liver, and brain. It is the rate-limiting step of glycogen breakdown. Heinz bodies is correct, but both answers in this choice are not correct E. Incorrect- Pyruvate Dehydrogenase deficiency would result in an accumulation of lactic acid, alanine, and pyruvate. The signs and symptoms are not present here. Negri bodies are an indication for the Rabies virus and can be found in neurons.

A 25 year old female presents to the clinic with anemia, splenomegaly, and jaundice. She has an inherited disorder that reduces the assembly of the red cell skeleton and destabilizes the plasma membrane. Which of the following disorders does she present with? A. Hereditary Spherocytosis B. Paroxysmal Nocturnal Hemoglobinuria C. Immunohemolytic Anemia D. Thalassemia E. Sickle Cell Disease

Correct Answer: A. Hereditary Spherocytosis is caused by intrinsic defects in the red cell membrane skeleton that render red cells spheroid, less deformable, and vulnerable to splenic sequestration and destruction. Mutations that involve a-spectrin, b-spectrin, ankyrin, band 4.2, or band 3 weaken the interactions between these proteins, which cause red cells to lose membrane fragments as they age. To accommodate the resultant change in the ratio of surface area to volume, these cells adopt a spherical shape. A. Correct - see above B. Incorrect - results from mutations in the PIGA gene, which is essential for the synthesis of certain membrane associated complement regulatory proteins C. Incorrect - caused by antibodies that recognize red cells and lead to their premature destruction D. Incorrect - caused by the decreased synthesis of a-globin or b-globin, leading to anemia, tissue hypoxia, and red cell hemolysis E. Incorrect - caused by a point mutation in b-globin that promotes the polymerization of deoxygenated hemoglobin

A 50 year old man returns from a business trip to a developing country, where bacterial diarrheal diseases are common. Upon returning to the United States, the man starts feeling symptoms of watery diarrhea and abdominal cramps 1-2 days later. The man is informed by his PCP that he has an Enterotoxigenic E. coli infection and most likely contracted it from his recent travel. How is this bacterial infection primarily acquired? A. Person-to-person spread B. Consumption of fecally contaminated food or water C. Contact with bodily fluids D. Through aerosols E. Through contact with an animal vector

Correct Answer: B ETEC is spread through consumption of fecally contaminated food or water. That is why it is common in developing countries where sanitary standards are not always maintained. A - Incorrect because this bacteria is not spread through person-to-person contact. If you chose this, you might have been thinking of STEC, which is spread through person-to-person contact. B - Correct - see Above C - Incorrect because the bacteria is present in fecal material, not bodily fluids. D - Incorrect because the bacteria is not spread through respiratory droplets. In fact, none of the Enterobacteriacea in the chapter are spread through aerosols. E - Incorrect, because this bacterial infection is not associated with contact with animals. If you chose this, maybe you were thinking of another Enterobacteriaceae that is spread through animal vectors such as Yersinia.

A 50-year old man with a history of heavy alcohol use has presented to the Emergency Department citing increased thirst, nausea, and fatigue. Upon further inspection, it is found that he has cirrhosis of the liver as a result of secondary hemochromatosis. Which of the following changes inside of his body can likely be attributed to this condition? A. Decreased erythroferrone B. Decreased hepcidin C. Decreased RBC progenitors D. Decreased iron absorption in the gut E. Increased RBC production

Correct Answer: B Explanation: Secondary hemochromatosis occurs as a result of ineffective erythropoiesis. In this condition, erythropoietin levels are increased causing an increase in RBC progenitors in the bone marrow (This means answer choice C is incorrect). An increased amount of the hormone erythroferrone is produced (Answer choice A is incorrect) and RBC production is defective (Answer choice E is incorrect). Erythroferrone is an inhibitory regulator of hepcidin, so hepcidin is decreased (Answer choice B is correct). Hepcidin is a negative regulator of iron absorption, so a decrease in its levels causes more iron to be absorbed in the gut, causing an iron overload and damage to various organs over time (Answer choice D is incorrect).

A 23-year-old male presents to the emergency department after training for a Marathon, with complaints of his urine being a red-brown color. Upon further examination it is also noted that the patient presents with signs of Jaundice. It is concluded that this patient is experiencing Intravascular hemolysis, what other clinical manifestations would you expect this patient to experience? A. Splenomegaly B. Renal Hemosiderosis C. Hyposthenuria D. Dactylitis in the hands and feet E. Hemoglobin Barts

Correct Answer: B - After Hemoglobin is released from RBCs, it is bound to haptoglobin. As haptoglobin is depleted, hemoglobin is oxidized to methemoglobin - this will get reabsorbed in the proximal tubule, but some is excreted in the urine causing the red-brown color. Hemoglobin will release Iron into tubular cells which causes the Renal Hemosiderosis. A. Incorrect - This is associated with Extravascular hemolysis, where destruction of the RBCs occurs in the spleen B. Correct - See Above C. Incorrect - This is when sickled cells damage the renal medulla, which leads to an individuals' inability to produce concentrated urine D. Incorrect - This is a form of Vaso-occlusive crises that cause severe pain to an affected region. This is commonly associated with sickle cell. E. Incorrect - Hemoglobin Barts is an a-thalassemia disorder within newborns, where excess unpaired y-globin is formed into tetramers.

A 52-year-old female with a history of Stage I colorectal cancer presented to the ED with complaints of neuropathy in bilateral upper extremities and pallor that has been ongoing for the past week and gradually becoming more prominent. She does have a surgical history of colon resection to remove the tumor and gastrectomy for other GI-related issues. Since her cancer diagnosis, she has only been eating vegetables, CBC was obtained that demonstrated anemia with HGB of 7.5. MCV was noted to be elevated at 112 fL/cell. Based on her clinical symptoms and lab findings, what is she likely deficient in and what is the main cause of her deficiency respectively? A. Folate Deficiency; Not eating meat B. Cobalamin Deficiency; Loss of parietal cell function and IF secretion C. Folate Deficiency; Loss of function of the mitochondrial enzyme methylmalonyl-CoA mutase D. Cobalamin Deficiency; Resection of the Colon E. Folate Deficiency; Loss of parietal cell function and IF secretion

Correct Answer: B - Given her MCV is elevated, that is indicative of macrocytic anemia. Since she does not have any history of advanced liver disease or severe hypothyroidism and has symptoms of neurologic abnormalities, this would allude to megaloblastic anemia. Considering that she is a vegetarian, folate deficiency can be ruled out as leafy vegetables are a great source of folate. Hence, the patient is likely deficient in cobalamin. Given her history of gastrectomy, this would impair the function of parietal cells and IF secretion. Intrinsic factor is important in the absorption of Vitamin B12. Without the secretion of intrinsic factor, this leads to malabsorption of Cobalamin leading deficiency of Cobalamin. A) Incorrect - Patient is a vegetarian and getting her source of folate from leafy vegetables, hence, she is not deficient in folate. Also, meat is not considered a reliable source of folate due to prolonged cooking B) Correct - See above for explanation C) Incorrect - Although the loss of function of the mitochondrial enzyme methylmalonyl-CoA mutase may be the cause of her neurologic symptoms, Cobalamin is a cofactor of this enzyme, not folate D) Incorrect - Patient is deficient in cobalamin, however, the resection of colon does not affect the absorption of cobalamin, rather, resection of the terminal ileum would. E) - Incorrect - Although the loss of function of parietal cells and IF secretion can lead to megaloblastic anemia, patient is not likely to develop folate deficiency given that she is a vegetarian eating leafy vegetables

A 44-year-old male patient who has been diagnosed with diabetes mellitus for 6 months visits his primary care physician for a follow-up appointment regarding his blood work. The male insists he has been limiting his sugar intake but has not been consistent with his diabetes medication, Glucophage. Which of the following elevated levels in the patient's blood work reveals that his diabetes has not been effectively controlled? A. Bilirubin B. HbA1c C. Thrombocytes D. HbF E. Erythrocytes

Correct Answer: B - HbA1c is one of the subtypes of hemoglobin type HbA which binds irreversibly to glucose. Glycosylated hemoglobin is important in monitoring an individual's glucose levels because this hemoglobin can only be disposed of once the red blood cells contained are terminated; since erythrocytes have a lifespan of about 120 days, it is a good indication of an individual's blood glucose levels over the span of 4 months. In a healthy individual or in a patient whose diabetes is controlled, HbA1c levels should not exceed 7% of the total hemoglobin. If this number exceeds, it is clear that the individual has had high glucose levels for at most 4 months. A. Incorrect - Bilirubin is a waste product from the breakdown of the heme moiety of the hemoglobin molecule which occurs during the degradation of senescent erythrocytes. High levels of bilirubin indicate jaundice. B. Correct- see above. C. Incorrect - Thrombocytes, or platelets develop from megakaryocytes and help to form blood clots to lessen and eventually stop bleeding. Elevated thrombocyte levels indicate thrombocytosis. D. Incorrect - HbF, or fetal hemoglobin accounts for less than 1% of the total hemoglobin in adults and is the predominant form of hemoglobin in the fetus. Deviation of normal HbF levels may appear in those with sickle cell disease or thalassemia but there is no pathological role in the disease. E. Incorrect - Erythrocytes, or red blood cells levels cannot indicate blood glucose levels. Hematocrit levels demonstrate the volume of red blood cells in a sample of blood. Inflation of normal hematocrit levels, or high levels of circulating erythrocytes, can indicate polycythemia vera or dehydration.

A 36-year-old female goes to her General Family Physician after complaining of feeling tired and short of breath during the week. She complains of difficulty concentrating and frequent headaches and having a hard time falling asleep although she cuts caffeine in her daily intake. Upon physical examination, the doctor noticed that the patient's skin was pale and so the doctor ordered for blood to be drawn and sent to the laboratory for CBC. Once the results arrived, it illustrated the patient has a low hemoglobin concentration of 10.3g/dL and further laboratory results showed that the patient had a low vitamin B12 as well. The result illustrates ... A. Sickle Cell disease B. Anemia C. Barr body D. Neutrophilic E. Leukocytes

Correct Answer: B- A person who tend to suffer from Anemia tend to showcase these types of symptoms such has have a low Hemoglobin concentration and feeling tired and short of breath Also, symptoms such as difficulty concentrating, frequent headaches, pale skin, leg cramps, and insomnia. A.Incorrect- The patient doesn't suffer due to a mutation which impacted the shape of a normal blood cell which becomes a biconcave disc shape which presents different symptoms. B.Correct- see above C.Incorrect- not relevant to the question because it is the condensed, single inactive X chromosome which forms a drumstick shaped appendage on one of the nuclear lobes D.Incorrect- since neutrophils are one type of the WBC which perform their immune functions of patrolling for and eliminating pathogens. E.Incorrect- White blood cells which are the cells of the immune system that are involved in protecting the body against both infectious disease and foreign invaders.

A 30 y.o african male presents to his primary care physician with a fever, dark urine, fatigue, and pale skin. The patient explains to his physician that he started taking chloroquine, an antimalarial medication, in preparation for his trip to Asia, and it was shortly after that he started experiencing these symptoms. Physical examination is remarkable for splenomegaly. The clinical workup shows evidence of hemolysis, increased bilirubin, and anemia. Blood smear findings include darkly stained regions within the red blood cells also known as Heinz bodies. The physician suspects that his symptoms are being caused by a mutation in a gene. What is the most likely diagnosis in the patient? A. Niacin deficiency B. Glucose 6 phosphate dehydrogenase deficiency C. B12 deficiency D. Thiamine deficiency E. Increased generation of glutathione

Correct Answer: B- G6PD is the catalyst in the rate limiting first step of the PPP, which uses G6P to convert NADP into its reduced form, NAPHD. In RBCs, NADPH is critical in preventing damage to cellular structures caused by oxidative stress. It does this by serving as a substrate to the enzyme glutathione reductase. Reduced glutathione can be used to convert hydrogen peroxide to water and prevent damage to cellular structures, particularly the cell wall of the RBCs. G6PD deficiency is an X-linked recessive disorder so its more common in males. A deficiency in this enzyme makes RBCs more susceptible to damage by free radicals, which leads to hemolysis. This deficiency can also result in episodic hemolytic anemia. Also the Heinz bodies in the blood smears are very specific to this condition! A. Incorrect- this is incorrect because niacin deficiency occurs when a person doesn't get enough or can't absorb niacin or its amino acid precursor, tryptophan. Niacin deficiency results in pellagra. B. Correct- see above C. Incorrect- B12 deficiency is due to the lack of intrinsic factor produced by the parietal cells in the stomach. D. Incorrect- decreased transketolase activity is responsible for a deficiency in thiamine. E. Incorrect- the PPP yields NADPH, which is essential for converting oxidized glutathione back to its reduced form. Reduced glutathione is capable of neutralizing ROS and free radicals and therefore protecting RBCs from oxidative damage. In the absence of reduced glutathione (due to G6PD deficiency), RBCs become more susceptible to oxidative stress, resulting in hemolysis.

A patient is seen in the clinic showing signs of fatigue, paleness, jaundice, and dark urine. Upon collection of a blood smear, Heinz bodies were noted. If the patient is suspected to have glucose-6-phosphate dehydrogenase deficiency, symptoms are occurring directly due to reduced production of which of the following? a. NADH b. NADPH c. ATP d. FADH2 e. O2

Correct Answer: B. NADPH is a byproduct of the pentose phosphate pathway. When glucose-6-phosphate becomes 6-phospho-gluconate via glucose-6-phosphate dehydrogenase, it will convert NADP+ to NADPH and H+. a. Incorrect- NADH is produced from glycolysis not the pentose phosphate pathway. b. Correct- See above c. Incorrect- ATP is produced from glycolysis not the pentose phosphate pathway. d. Incorrect- FADH2 is produced from citric acid cycle, not the pentose phosphate pathway. e. Incorrect- O2 is not a byproduct of the pentose phosphate pathway.

A 6-year-old boy is experiencing severe delay in development of motor skills and suffers from hypotonia. Further testing determines the child has an accumulation of undegraded molecules building up in lysosomes. The lysosomes have become nonfunctional and form inclusion bodies because of a lack of which marker on the glycoprotein? a. N-acetylglucosamine b. Mannose-6-phosphate c. N-acetylneuramic acid d. Mannose e. phosphotransferase

Correct Answer: B. The boy is experiencing Inclusion cell (I-cell) disease, a rare condition in which lysosomal enzymes lack the mannose phosphate marker that targets them to lysosomes. A lack of this marker causes the lysosomes to be nonfunctional and molecules cannot be degraded. a. Incorrect-It is a monosaccharide in salivary mucin not lysosomes b. Correct c. Incorrect-A major component in salivary mucin d. Incorrect-A precursor to mannose-6-phosphate in lysosomes e. Incorrect-This is the enzyme that is defective in I-cell disease that causes the lack of mannose-6-phosphate

A Caucasian female of approximately 20 years of age arrives to the ER via ambulance. The EMTs inform the trauma surgeon on call that she was an innocent bystander at a college party where someone pulled a gun, unfortunately for her she took a bullet to the upper leg. She sustained heavy blood loss but fortunately the bullet passed through without impacting any bones or major nerves. Which of the following metrics would you expect to be greater than normal 3 days post-op? A. Intravascular Volume B. Hematocrit C. Reticulocyte Count D. RBC Count E. Platelet Count

Correct Answer: C Immediately following massive hemorrhaging you'd expect to see a large reduction in intravascular volume which would include RBCs, leukocytes, platelets, reticulates, and most other circulating components of blood. However following the traumatic event, during the bodies attempt to return to homeostasis, you would see a massive increase in reticulocyte production from the bone marrow (approx. 10%-15% at 7 days). Therefore, the choice of C makes the most sense a week out from the event. A. Incorrect because most of the effects of acute blood loss are directly related to the intravascular volume decreasing B. Incorrect because the hematocrit would decrease due to hemodilution triggered by movement of fluid from ISF into the intravascular compartment C. Correct, see above for explanation D. Incorrect because of hemodilution explained in B E. Incorrect because of hemodilution explained in B

A 54-year-old male reports to his new PCP with complaints of fatigue, weakness, and chest pain. In the appointment after reviewing the man's chart, the doctor finds that he had a transcatheter aortic valve replacement about a year ago. Suspecting the man to have anemia the physician orders for a blood smear to confirm. What cells in addition to RBCs would be found in the blood smear? A. Bite Cells B. Sickled Cells C. Burr/Helmet Cells D. Spherocytes E. Heinz Bodies

Correct Answer: C This is a case of anemia resulting from trauma to RBCs. There are two ways in which this can happen, most instances are associated with cardiac valve prostheses, as seen in this case, and secondly in microangiopathic disorders such as DIC, TTP, and HUS. Regardless, both involve the creation of microvascular lesions that result in luminal narrowing producing sheer stresses that mechanically inure the RBCs. In blood smears you will see cell fragments (schistocytes) also referred to as "Burr Cells" or "Helmet Cells" A. Incorrect: Bite cells are characteristic of G6PD deficiency, Bit cells form when the phagocytes in the spleen pluck out Heinze bodies from RBC B. Incorrect: Sickled cells are characteristic of Sickle Cell Disease which involves a point mutation in the Beta-globin. C. Correct: See Above D. Incorrect: Spherocytes are often found when there are problems with the RBC cell membrane proteins and involved mostly in extracellular hemolysis. E. Incorrect: Heinz bodies form due to crosslinking of reactive sulfhydryl groups on globin chains, which is caused by high levels of oxidants due to a G6PD deficiency

A 29 year old female goes to the doctor to find out what her blood type is. She wants to see if she could donate blood to a sibling needing a transfusion. They tell her she has a B antigen and has anti-A serum antibodies. What blood type does the sibling need to be in order to be able to receive her blood? A. Rh negative B. A C. AB D. O E. Rh positive

Correct Answer: C - A person with Type B blood can give blood to a person with Type AB because they possess the B antigen. A. Incorrect, there is not enough information given in the question to know if the person is Rh- B. Incorrect, A person with type B blood cannot give blood to a person with Type A because they do not possess the type B antigen C. Correct, see above D. Incorrect, A person with type B blood cannot give blood to a person with Type O because they do not possess the type B antigen E. Incorrect, there is not enough information given in the question to know if the person is Rh+

A 62-year-old male presents to the ED complaining of fever (101.5 degrees F), abdominal pain, and edema (swelling) to his lower extremities, most notably to the ankles and feet. The patient notes the edema is worse at the end of the day and admits to a history of alcohol abuse. The ER physician suspects liver damage and orders a series of blood tests including a liver function test (LFT). The results of the liver function test were abnormal including low plasma protein levels. What is the main protein constituent of plasma that causes swelling of the ankles due to lowered colloid osmotic pressure? A) Immunoglobulins B) Fibrinogen C) Albumin D) Glucose E) Urea

Correct Answer: C - Albumin is the main protein constituent of plasma and accounts for approximately half of the total plasma proteins. The textbook reveals albumin is made in the liver and responsible for "exerting" the concentration gradient between the ECF and our blood. If large amounts of albumin leak out of blood vessels and into CT or is lost through kidney excretion, colloid osmotic pressure decreases and fluid accumulates in the tissues causing edema. The textbook specifically states the increase in tissue fluid is noted by swelling of the ankles towards the end of the day due to the "responsibilities" of albumin. A - Immunoglobulins, are globulins, which are constituents of plasma protein, however, it is not the main protein constituent. Immunoglobulins are antibodies and have an essential role in the immune system not with maintaining osmotic pressure. Nonimmune globulins help maintain osmotic pressure, however, the answer choice specifically states immunoglobulins. B - Fibrinogen, is also a constituent of plasma protein, however is not the main plasma protein constituent. Fibrinogen is transformed into fibrin and its main role is to maintain normal blood clotting not colloid osmotic pressure. C - See Above D - Glucose, is a component of blood plasma, however, it is a nutrient not a plasma protein. E - Urea, is a component of blood plasma, however, it is a nonprotein nitrogen substance not a plasma protein.

A 29 year old male presents to his primary care physician for a scheduled complete blood count (CBC) following a previous annual visit. He has a history of having unprotected sex with multiple partners over the past 6 months and has not traveled outside of the country in the last year. His CBC results come back and show there is a decrease in his Leukocyte count with normal Erythrocyte, Hematocrit and Hemoglobin levels. Which of the following is NOT a probable cause of the decrease in his leukocytes? Chemotherapy/Radiation Bone Marrow Diseases Bone Fractures Autoimmune Disorders HIV/AIDS

Correct Answer: C - Bone Fractures, infection, inflammation, pregnancy or strenuous exercise would cause an elevation in Leukocytes (Hyperleukocytosis) Incorrect Answers: A, B, D, E - all cause a decrease in Leukocytes (Leukopenia)

A 35-year-old female presents to the clinic with unexplained bruising, shortness of breath, fatigue and an abnormal discoloration of her urine close to when she wakes up in the morning. A flow cytometry detects the absence of GPI-linked proteins therefore, diagnosing the paroxysmal nocturnal hemoglobinuria. What makes red blood cells deficient in GPI-linked factors more prone to lysis? A. Damaged bone marrow B. A point mutation in β-globin C. Genetic mutations that inactivate PIGA D. Deficiency in the uptake of vitamin B12 E. Deficient globin synthesis

Correct Answer: C - The correct answer is C because paroxysmal nocturnal hemoglobinuria is characterized by the acquired genetic mutations in phosphatidylinositol glycan complementation group A gene (PIGA) which inactivates the GPI-linked proteins in RBCs allowing them to lyse prematurely or easily lyse due to injury. A. Incorrect - Aplastic anemia is characterized by damaged bone marrow that is unable to make enough red blood cells. B. Incorrect - This disorder is not due to an inherited blood disorder; Sickle cell anemia can be classified by this. C. Correct - See above. D. Incorrect - Anemias such as pernicious anemia is caused by an intrinsic factor that has been impaired and no longer allows for the normal uptake of vitamin B12 from the gut. E. Incorrect- Deficient globin synthesis is due to a hemoglobin abnormality that is characteristic of thalassemia syndromes.

A 34-year-old male present to the ED with complaints of fatigue, difficulty breathing, and pallor. His radial pulse is rapid and easily palpable bilaterally. The patient reveals he is on day 6 of 10-day course of penicillin to treat a recent bout of strep throat. CBC results revealed reticulocytosis and peripheral blood smear demonstrates presence of spherocytes. A direct Coombs test confirmed warm hemolytic anemia, suspected to be drug-induced. Through which mechanism did the penicillin induce the hemolytic anemia? A. Folate antagonist mediated B. IgM mediated C. Hapten mediated D. Rh-antibody mediated E. Spectrin mediated

Correct Answer: C - Warm hemolytic anemia is idiopathic but is associated with autoimmune disease, lymphoproliferative disorders, or drugs. In the case of drug-induced hemolysis, there are several mechanisms through which drugs may induce auto-immune hemolytic anemia. Penicillin causes auto-immune hemolytic anemia by acting as a hapten once bound to erythrocytes, meaning it elicits an immune response upon binding to a larger carrier (i.e. erythrocytes). Antibodies attack the hapten (i.e. penicillin) and so hemolysis only occurs in the presence of the drug. A. Incorrect— Methotrexate and sulfa drugs are direct folate antagonists that mimic folate deficiency and cause megaloblastic anemia. Peripheral blood smear would demonstrate hypersegmented neutrophils. B. Incorrect— IgM antibodies are associated with cold hemolytic anemia. C. Correct—see above D. Incorrect— Associated with drugs Methyldopa, Levodopa, Procainamide, Ibuprofen, Interferon-α (see table 47-6) E. Incorrect— Hemolytic anemia arising from spectrin mutation is the result of a genetic disorder known as hereditary spherocytosis.

An 18-year-old male presents to the urgent care with fatigue, dizziness and a headache. Physical examination noted pale skin, jaundice and splenomegaly. Blood work was ordered, and the results are consistent with hemolytic anemia. Defects in major erythrocyte cytoskeleton proteins are known to lead to hemolytic anemia. Which of the following is NOT a major protein of this cytoskeleton? a. Spectrin b. Actin c. Transferrin d. Ankyrin e. Band 4.1

Correct Answer: C. Transferrin- Transferrin is a blood-plasma glycoprotein and plays a central role in iron metabolism. It is responsible for ferric-ion delivery. a. Spectrin- Spectrin is the most abundant protein in the erythrocyte cytoskeleton and is composed of alpha- and beta-subunits wound to each other. b. Actin- Each actin filament binds multiple spectrins, resulting in a branched membrane cytoskeleton c. Transferrin- see above d. Ankyrin- Ankyrin connects the spectrin cytoskeleton to the membrane lipid bilayer, by interacting with beta-spectrin & band 3. e. Band 4.1- Band 4.1 anchors the spectrin skeleton with the membrane by binding glycophorin C and the actin complex

A 25-year-old, African American male presents to the Emergency Department with severe pain in his bones. It was later discovered that he was suffering from a vaso-occlusive pain crisis. His blood smear indicates needlelike fibers within red cells, and a lab evaluation shows increased MCHC levels which facilitates the formation of these needlelike fibers. Which of the following would be the most likely cause of this disease? A. G6PD Deficiency B. Frameshift mutation affecting spectrin C. Point mutation in β-globin D. Chain terminator mutations E. Absent synthesis of α-globin chain

Correct Answer: C—The patient comes in with sickle cell anemia. The key factors in this differential diagnosis are that the patient is African American, he presents with a vaso-occlusive pain crisis, and the increased MCHC levels which facilitates the sickling of the red blood cells. The cause of sickle cell anemia is the point mutation of β-globin. This point mutation promotes the polymerization of deoxygenated hemoglobin which leads to the red cell distortion. Incorrect—This causes G6PD related hemolytic anemia Incorrect—This causes Hereditary Spherocytosis Correct—see above Incorrect—This causes βo-Thalassemia Incorrect—This causes α-Thalassemia

A five year old girl of African descent presents to the emergency department with a low grade fever, cough, and chest pain. An x-ray is performed showing bilateral pulmonary infiltrates. The parents mention that the patient was diagnosed with sickle cell anemia at 8 months old. What factor is most likely increasing the rate of RBC sickling, leading to her symptoms? A. Increased concentration of HbF B. The patient is heterozygotic for the HbS/HbC gene C. Low MCHC D. Decreased transit time of RBC's through microvascular beds E. Increased intracellular pH

Correct Answer: D A. Increased concentration of HbF ○ Incorrect: HbF expression usually falls around 5-6 months of age therefore, an individual with increased concentration of HbF would have a condition called Hereditary persistence of Fetal Hemoglobin, a less severe form of sickle cell anemia. B. The patient is heterozygotic for the HbS/HbC gene ○ Incorrect: HbS/ HbC heterozygotes typically have a milder form of sickle cell disease called HbSC Disease. C. Low MCHC ○ Incorrect: Increased MCHC will facilitate sickling of RBC's, therefore, low MCHC will decrease the rate of sickling and severity of the disease. D. Decreased transit time of RBC's through microvascular beds ○ Correct. The patient is experiencing acute chest syndrome, a vaso-occlusive crisis in the lungs, a complication of sickle cell disease. Pulmonary inflammation slows the movement of RBC's through the microvasculature due to adhesion of leukocytes to endothelial cells, making inflamed vascular beds more prone to sickling and occlusion. E. Increased intracellular pH ○ Incorrect: Decreasing intracellular pH would increase the rate of sickling due to an increased fraction of deoxygenated HbS.

A 65 year old man presents to the ED due to chronic fatigue, pale skin, and shortness of breath. The patient had been recently discharged from the hospital a few days prior due to a remission of a malignant tumor. The physician on site suspects a nutritional deficiency and runs a blood test. Results indicate normal WBC and platelet count but a marked reduction in red blood cells. Suspecting anemia, the physician performs a bone marrow biopsy and notes an abnormally high ratio of adipocytes to hematopoietic cells. Which of the following disease conditions below best describes the patient's affliction? A. Hypercellular Bone Marrow B. Pernicious Anemia C. Acute Myelogenous Leukemia D. Aplastic Anemia E. Polycythemia

Correct Answer: D - Aplastic anemia is a disease condition marked by only a small number of blood-forming cells present in a marrow sample. Given that one of the primary causes of this condition is the use of chemotherapeutic drugs, the patient's history of cancer acts as supporting evidence. In addition, the 65 year old patient denotes the fact that bone cellularity decreases with age, thereby causing hypocellular bone marrow which is directly associated with aplastic anemia. A. Incorrect- This condition is marked by hematopoietic tumors similar to hypocellular bone marrow, which is related to aplastic anemia. However, this condition is marked by a high ratio of red blood cells to adipose tissue. The bone marrow aspiration presented the opposite ratio B. Incorrect- Although it is a form of anemia and is similarly marked by a reduction in red blood cells, the causative agent is a vitamin B12 deficiency. Given the blood test did not show a decrease in folate, it can be ruled out C. Incorrect- This is a subclassification of hypercellular bone marrow, which again is a marked increase in red blood cells D. Correct- See above E. Incorrect- Despite being a red blood cell cancer, this disease condition is when the bone marrow produces too many red blood cells

During the check in process at the clinic, a patient responds to the questionnaire by saying he had recently traveled to Eastern Europe. Before walking into the patient's room, the doctor glances at the patient's recent lab work and sees that all of his white blood cell counts are elevated. He remembers there is a new species of helminths that are rampant in Eastern Europe and begins to wonder if that is the cause of the patient's abnormal blood counts. Blood cells involved in protection from parasites, such as worms, are also involved in which of the following activities in the body? a) Degradation of microbial structures b) Smooth muscle contractions c) Initiation of the respiratory burst d) Asthma and allergy responses e) Antibody secretion

Correct Answer: D - Asthma and allergy responses. This is the correct answer because eosinophils are responsible for protection from worms. Eosinophils contain cationic enzymes and hydrolytic enzymes in their granules, which are used to destroy parasites. Eosinophils are also present in the responses carried out against asthma and allergies. Basophils are also involved in the allergic response, however not in the asthma response, making this the best answer. a) Degradation of microbial structures is carried out by basophils. They carry enzymes such as proteases in their granules that allow them to degrade microbial structures b) Smooth muscle contractions are also a consequence of basophil activation. This activation causes the release of histamine, inducing contractions in smooth muscles c) Neutrophils are responsible for the respiratory burst, a process by which free oxygen radicals are formed for the purpose of destroying foreign materials/organisms found at the site of infection d) Correct: see above e) Antibody secretion is done by agranulocytes, specifically B-cells, which secrete antibodies in response to antigen binding.

A 58-year-old male accountant arrives at the doctor for his routine physical. He offers complaints of fatigue and feeling lightheaded. He attributes this to working extra hours at a local gun range. What is an accurate explanation regarding his condition? a) Decreased δ-ALA, decreased protoporphyrin IX b) Increased Porphobilinogen, increased Copropophyrinogen III c) Decreased δ-ALA, increased protoporphyrin IX d) Increased δ-ALA, increased protoporphyrin IX e) Decreased Porphobilinogen, increased Copropophyryinogen III

Correct Answer: D - During lead poisoning, both δ-ALA and protoporphyrin IX accumulate during the synthesis of heme which leads to a decreased production of heme. The accumulation of δ-ALA is due to the negative feedback loop on δ-Aminolevulinic acid synthase from heme. Therefore, this patient has anemia which explained his fatigue and lightheadedness. His extra hours working at the gun range indicates that he has been dealing with lead. a) Incorrect - Both δ-ALA and protoporphyrin IX increase, not decrease for lead poisoning. b) Incorrect - These molecules, Porphobilinogen and Copropophyryinogen III do not accumulate in this pathway due to lead poisoning. c) Incorrect -δ-ALA does not decrease due to lead poisoning. d) Correct: see above e) Incorrect - These molecules, Porphobilinogen and Copropophyryinogen III do not accumulate in this pathway due to lead poisoning.

A 25 year old male presents to the Emergency Department with generalized weakness and fatigue. After an abnormal CBC, the physician orders a blood smear which reveals spherical erythrocytes. The patient is diagnosed with hereditary spherocytosis, a genetic disorder that causes an abnormal shape in erythrocytes. What is the function of the defective protein complex in erythrocytes? Express specific blood group antigens on the surface of erythrocytes Transport oxygen and carbon dioxide throughout the circulatory system Maintain the colloid osmotic pressure to prevent erythrocyte swelling Interact with spectrin filaments to anchor the plasma membrane to the cytoplasm Bind hemoglobin and act as an anchoring site for cytoskeletal proteins

Correct Answer: D - Hereditary spherocytosis is an autosomal dominant genetic disorder affecting ankyrin protein complex. This complex of proteins interacts with spectrin, band 3, and other erythrocyte-integral membrane proteins, and functions in anchoring the erythrocyte plasma membrane to the cytoplasm. Defects in this protein complex cause the plasma membrane to detach from the cytoplasm resulting in spherical erythrocytes. Incorrect - Extracellular domains on integral membrane proteins are glycosylated and express blood group antigens (A, B, and O). These antigens determine the four primary blood groups. These antigens do not function to maintain the shape of erythrocytes Incorrect - Oxygen and carbon dioxide are transported via erythrocytes bound to the protein hemoglobin. Hemoglobin binds oxygen in the lungs and unloads it in the tissues. Incorrect - Albumin is the main protein constituent of plasma, not erythrocytes, and is responsible for exerting the concentration gradient between blood and extracellular tissue fluid. Correct - See above Incorrect - B and 3 protein is the most abundant transmembrane protein in the erythrocyte cell membrane. Its main function is to bind hemoglobin and act as an anchoring site for cytoskeletal proteins. The ankyrin protein complex binds to Band 3 protein, but Band 3 does not directly function to maintain the shape of the erythrocyte.

A 5-year-old female child presents to her primary care physician as her parents are concerned about her being unusually fatigued over the past few days. Upon physical examination, it is noted that the child is pale and presents with bilateral icterus. The parents indicate that the child is adopted, and the family history of the birth parents is unknown. Lab evaluation and molecular analysis indicate hemolytic anemia caused by a disorder that is inherited in an autosomal recessive fashion. The disorder results in the mutation of an enzyme that leads to an increase in 2,3-BPG levels in the erythrocyte as a compensating mechanism. A deficiency in which of the following enzymes would explain the child's symptoms? A. Cytochrome b5 Reductase B. Delta - ALA Dehydratase C. Glucose 6-PD D. Pyruvate Kinase E. Ferrochelatase

Correct Answer: D - Pyruvate kinase deficiency is an inherited autosomal recessive deficiency that results in hemolytic anemia. This deficiency leads to an elevation in 2,3-BPG concentration as a result of the blockage of the conversion of phosphoenolpyruvate to pyruvate. This acts as a compensating mechanism and the effects of the anemia are frequently moderated because 2.3-BPG decreases the affinity of hemoglobin for oxygen and allows the remaining red blood cells to be efficient in releasing the bound oxygen to tissues. A. Incorrect- Leads to congenital methemoglobinemia, the presence of excess methemoglobin B. Incorrect- An enzyme in the heme synthesis pathway, deficiency leads to delta - ALA Dehydratase porphyria C. Incorrect- Although Glucose 6-PD deficiency also results in hemolytic anemia, it is inherited in an X-linked pattern and occurs more frequently in males. D. Correct - See above E. Incorrect- An enzyme in the heme synthesis pathway, deficiency leads to erythropoietic protoporphyria

A 2-year old male child presents to the clinic with symptoms of Tay Sach's Disease. However, the observed symptoms appear to be of a progressed stage of the disease. Enzyme assay reveals inactive hexosaminidase A, consistent with the initial diagnosis, but also indicates that hexosaminidase B is inactive in the child. What is responsible for the acceleration of the disease state in this child? A. Mutation of the HexA gene, resulting in an accumulation of ganglioside GM2 B. Mutation of the HexB gene, resulting in an accumulation of ganglioside GM2 C. Mutation of the HexA gene, resulting in an accumulation of ganglioside GM2 and globoside D. Mutation of the HexB gene, resulting in an accumulation of ganglioside GM2 and globoside E. Mutation of the hexosaminidase activator protein

Correct Answer: D - Sandhoff disease shows a clinical course that is similar to that of Tay Sach's Disease, however Sandhoff disease has an accelerated timetable. Sandhoff disease results from a mutation to the HexB gene, which generates a defective Beta subunit, ultimately resulting in the inactivation of both hexosaminidase A and B. This accelerates the disease's timetable by causing an accumulation of both ganglioside GM2 and globoside. A. Incorrect - this describes the mutation that causes Tay Sach's disease B. Incorrect - mutation to the HexB gene will inactivate both hexosaminidase A and B. Without Hexosaminidase B functioning properly, globoside cannot be cleaved and will also accumulate. C. Incorrect - mutation of the HexA gene will only result in an inactive hexosaminidase A. With hexosaminidase B still active, globoside will be cleaved and will not accumulate. D. Correct - see above E. Incorrect - In Sandhoff Activator Disease, the protein responsible for activating hexosaminidase A is mutated. This results in a decrease, but not total loss, of hexosaminidase A function. Furthermore, this will have no effect on hexosaminidase B function.

A medical student presents to the primary care physician with complaints of general malaise and fatigue. She reports that yesterday she had been out drinking when she accidentally walked into a closed glass screen door and got a bloody nose that lasted about 30 minutes. She was celebrating because she had just finished taking an exam that she had studied extremely hard and had stressed over for about a week. The physician orders blood work and determines that the patient is suffering from chronic blood loss. What is most likely the cause of the blood loss and what test would the physician use to confirm the diagnosis? A: nosebleed and peripheral blood smear B: peptic ulcer and peripheral blood smear C: nosebleed and stool sample D: sickle cell anemia and peripheral blood smear E: peptic ulcer and stool sample

Correct Answer: E A: nosebleed and peripheral blood smear Incorrect. Because a nosebleed would not lead to chronic blood loss, but rather it would lead to acute blood loss . B: peptic ulcer and peripheral blood smear Incorrect: peptic ulcer is the correct cause, but a peripheral blood smear would only show the compensatory results of the chronic blood loss such as microcytic RBCs. It would be no help in determining the cause C: nosebleed and stool sample Incorrect: a nosebleed would not lead to chronic blood loss, but rather it would lead to acute blood loss D: sickle cell anemia and peripheral blood smear Incorrect: sickle cell anemia would not lead to chronic blood loss E: peptic ulcer and stool sample Correct: GI laceration is the most common cause of chronic blood loss and a peptic ulcer is the most common cause of a GI laceration. This is confirmed using a stool sample to see if there is blood present in the stool. (Peptic ulcer caused by stress)

Upon arrival to the ED, an 11-year-old male underwent anaphylaxis. His mother reported that she brought him to the ED due to an acute onset of hives on his left upper arm and difficulty breathing immediately after he was stung by a bee while climbing a tree with his friends. She stated that the patient had not been stung by a bee before until recently. The patient's vitals are as followed: BP 85/60 mmHg, HR 120 bpm, RR 32 breaths/min, and body temperature 98.6 ° F. Which of the following leukocytes is responsible for this patient's symptoms? A. Neutrophils B. Lymphocytes C. Eosinophils D. Monocytes E. Basophils

Correct Answer: E - During an allergic reaction, an antibody secreted by plasma cells known as IgE binds to the Fc receptors on basophils. This binding triggers basophils to release vasoactive agents from cell granules which are associated with hypersensitivity reactions and anaphylaxis (including rash/hives, hypotension, increased heart rate, difficulty breathing, and increased respiratory rate). A. Incorrect - Neutrophils are associated with inflammation, infection, phagocytosis, and tissue damage/wound healing but not allergic reactions and anaphylaxis. B. Incorrect - Lymphocytes are associated with the lymphatic system, inflammation, and wound healing and are the main functional cells of the immune system. They are not associated with allergic reactions and anaphylaxis. C. Incorrect - Eosinophils are associated with chronic inflammation, parasitic infections, wound healing, and allergic reactions, making this choice a possible answer. However, eosinophils are not associated with anaphylaxis making this answer choice incorrect. D. Incorrect - Monocytes are associated with the mononuclear phagocytotic system, inflammation, and wound healing but not allergic reactions and anaphylaxis. E. Correct - See above

A 6 month-old male presents to the clinic because his parents have noticed he has been having difficulty eating, crawling and remaining in an upright position. The physician decided to run a serum test to measure enzymatic activity. The results showed no hexosaminidase A and hexosaminidase B activity. What type of glycolipid(s) are expected to be accumulated in lysosomes? A. GM2 ganglioside B. Globoside C. GM1 ganglioside D. Ceramide E. GM2 ganglioside and Globoside

Correct Answer: E- The 3 year-old male that presents to the clinic has Sandhoff disease. This type of disease is very similar to Tay-Sachs, but has a deficiency in the enzymes hexosaminidase A and B. Hexosaminidase A deficiency leads to an accumulation of GM2 ganglioside and a hexosaminidase B deficiency leads to an accumulation of globoside. Sandhoff disease does not have any correlation to an ethnicity whereas Tay-Sachs Disease has a high prevalence in the Eastern European Jewish population. A. Incorrect - An accumulation of just this ganglioside would be an indicator of Tay-Sachs Disease with a deficiency in hexosaminidase A B. Incorrect - An accumulation of this ganglioside results from a deficiency in hexosaminidase B C. Incorrect - An accumulation of this ganglioside results from a deficiency in GM1-Beta-galactosidase leading to generalized gangliosidosis D. Incorrect - An accumulation of this ganglioside results from a deficiency in ceramidase leading to Farber disease. E. Correct - See above

A 29 year old female received a blood transfusion during a busy night at the hospital and is now showing signs of yellowing of the skin and sclera of the eye, hypotension, and renal failure. What would be the primary cause of these symptoms? A. The patient has a decreased erythrocyte count. B. The patient has a build up of conjugated bilirubin. C. The patient has primary familial and congenital polycythemia D. The patient has physiologic jaundice. E. The patient was administered ABO-incompatible blood.

Correct Answer: E- The patient is suffering from hemolytic transfusion reaction from receiving an incompatible blood transfusion due to clerical error. This is leading to massive hemolysis of the transfused erythrocytes and has caused systemic complications such as hypotension and renal failure. Incorrect; decreased erythrocyte count is a symptom of anemia, it is not the primary cause in this case. Incorrect; build up of conjugated bilirubin is not the primary cause but is occurring due to inhibition or blockage at the bile duct system leading to jaundice. Incorrect; primary familial and congenital polycythemia would occur with increased erythrocyte production and would not align with symptoms presented Incorrect; this condition primarily affects newborns and would not occur following a blood transfusion. Correct; see above

23-year-old female presents to her PCP with complaints of pelvic pain, increased urge to urinate, pain with urination, and blood in the urine for 2 days. She reports no recent travel or GI symptoms. Urine culture and lab tests reveal lactose positive, indole positive, gram negative rods. She is diagnosed with a bacterial UTI and treated with antibiotics successfully. Which virulence factors make these bacteria particularly virulent in regards to causing UTIs? A. Adhesins and hemolysin HlyA B. K1 capsules C. STa and LT1 toxins D. Stx1 and Stx2 toxins E. LPS and urease

Correct Answer: The bacteria that is causing her UTI is E. Coli. Adhesins (primarily P pili, AAF/I, AAF/III, and Dr fimbria) allow E. Coli to bind to cells lining the bladder and upper urinary tract, preventing elimination of the bacteria in voided urine. Hemolysin HlyA lyses erythrocytes and other cell types, leading to cytokine release and stimulation of an inflammatory response. A. Correct - see above B. Incorrect - Strains of E. Coli with the K1 capsular antigen are associated with neonatal meningitis, not UTI C. Incorrect - STa and LT1 are exotoxins associated with ETEC and cause Traveler's diarrhea, not UTI D. Incorrect - Stx1 and Stx2 are exotoxins (cytotoxic Shiga toxins) associated with STEC and Shigella, which cause watery diarrhea followed by bloody diarrhea. These may progress to hemolytic uremic syndrome (HUS) but she has no GI symptoms. E. Incorrect - Proteus mirabilis has LPS and urease, and causes UTIs, but is lactose negative. Also, LPS is not associated with causing UTIs.

A 32 year old female presents to her PCP with fatigue, shortness of breath, and lightheadedness.She mentions she recently visited the ED two weeks ago due to a UTI and was treated with a cephalosporin. Upon examining the left upper abdomen, the physician notices moderate splenomegaly. The PCP suspects immunohemolytic anemia. Which of the following antibodies would be the most probable causative agent and under what temperature would it most stably bind to red blood cells? IgA; 37 degrees celsius IgG; 37 degrees celsius IgA; 4 degrees celsius IgG; 4 degrees celsius IgM; 37 degrees celsius

Correct Answer: The correct answer is choice B- IgG; 37 degrees celsius. Cephalosporin is an antigenic drug that binds to the red cell membrane and creates a new antigenic determinant which is recognized by antibodies. Specifically the recognized antibodies are of the warm antibody type. These classes of antibodies bind most stably to red cells at 37 degrees celsius and the most causative antibodies are of the IgG class. Choice A: This is incorrect because although IgA antibodies can bind to red blood cells under 37 degrees celsius, they are far less common than IgG. Because the question asked for the MOST probable causative agent, IgG would be a better answer than IgA. Choice C: Not only is IgA the less probable causative agent, but it also binds to red blood cells in 37 degrees celsius not 4 degrees celsius. Cephalosporin is an antigenic drug that activates the warm antibody type and not the cold antibody type. Choice D: The first part of the answer IgG is correct but the temperature should be 37 degrees celsius since IgG is part of the warm antibody type. Choice E: IgM also causes immunohemolytic anemia but it is classified as part of the cold agglutinin type meaning it binds to red blood cells at 4 degrees celsius.

A 24-year-old woman presents to her physician with complaints of fatigue and weakness for several months. She mentioned how lately she has had trouble walking up the stairs to her apartment on the third floor, which she used to be able to do easily. Her partner also noticed that lately she's been very pale. On physical examination the physician noticed conjunctival pallor, but an otherwise healthy appearance. Lab results would reveal a decrease in what type of hemoglobin? a. HbA (α2β2) b. HbA2 (α2δ2) c. HbA (α2β1) d. HbF (α2γ2) e. HbA2 (α2β2)

Correct answer: A Explanation: the fatigue and weakness are typical signs of anemia, with the conjunctival pallor being the main clue. Since anemia is a reduction in red blood cell mass and hemoglobin is the main component of red blood cells, it is expected that there will be a decrease in adult hemoglobin (HbA) in the patient. All of the values can be found in table 47.1, page 502 of the textbook. A: Correct -HbA makes up 95-98% of adult Hb B: Incorrect- HbA2 makes up only 1.5-3.5% of adult hemoglobin C: Incorrect- the structure of HbA is A2B2, not A2B1 D: Incorrect- HbF is the major hemoglobin in infants below 4 months of age E: Incorrect- that is the structure of HbA, not HbA2

A young patient presents to a PCP with marked jaundice and is complaining of fatigue, weakness and dyspnea on mild exertion. The physician orders a blood smear, as well as other laboratory tests. The lab tests reveal anemia and the blood smear shows anisocytosis and several dark-appearing spherocytes with no central pallor. Upon further testing and review of the patient's family history the patient is diagnosed with Hereditary Spherocytosis. What medical procedure will be most beneficial for this patient and will help revert the anemia long term? A. Nephrectomy B. Splenectomy C. Partial hepatectomy D. Blood Transfusion E. Subtotal Thyroidectomy

Correct answer - B A. Nephrectomy ● Incorrect; the kidney's do not serve a significant role in this disease. The kidneys secrete erythropoietin, if the production of erythropoietin is high enough it can sometimes counterbalance the anemia in a patient with hereditary spherocytosis so removal of the kidney would only decrease erythropoietin and could lead to increased anemia. B. Splenectomy ● Correct. In Hereditary Spherocytosis the red blood cells (RBCs) have mutations in the proteins of the membrane skeletal leading to a small, spherical and less deformable structure of the RBC compared to normal RBCs. Normal RBCs have a deformable shaping ability which helps them be cleared from the body in the sinusoids. RBCs in hereditary spherocytosis get stuck in the splenic cords, due to their mutations, where they are easily ingested and degraded by macrophages. If you remove the spleen (splenectomy) then you can prevent these RBCs from getting stuck in the spleen and phagocytized. This will help prevent anemia, but the spherocytes will still exist. C. Partial hepatectomy ● Incorrect; the liver does not serve a significant role in this disorder and would not improve the patient's symptoms D. Blood transfusion ● Incorrect; a blood transfusion would temporarily revert the anemia in this patient but since this is a hereditary disorder the anemia would persist as the RBCs being produced by the patient would continue to carry the protein mutations and would keep getting stuck in the spleen and phagocytized leading to persistent anemia. This would only be a short-term solution used to support the patient during acute phases of anemia. E. Subtotal thyroidectomy ● Incorrect; The thyroid gland is not involved in the disorder and its removal would not improve this patient's symptoms.

A 30-year-old male presents to the emergency department with complaints of fatigue, generalized weakness, and shortness of breath. He recently started seeing a new primary care physician after moving to the area. He was diagnosed with sickle cell anemia as a child but has not had proper follow up since graduating from college. His prior diagnosis is likely causing his shortness of breath due to which of the following? A. Lack of spectrin proteins B. Inability of RBCs to maintain shape for proper oxygen transport C. Antibodies attacking RBCs D. Damaged RBCs blocking the airway E. Defective erythropoietin receptors

Correct answer - B: Sickle cell patients have sickle shaped RBCs. The biconcave disc shape serves to facilitate gas exchange across cell membranes a. Incorrect - the lack of spectrin proteins would cause a spherical shaped RBC (not sickle) b. Correct - see above c. Incorrect - this can occur when a patient receives a transfusion (causing a transfusion reaction - rare but deadly) d. Incorrect - Damaged RBCs that are no longer deformable become trapped in passages in the spleen where they are destroyed by macrophages e. Incorrect - this causes a higher-than-normal percentage of RBCs in circulation because receptors are not deactivated

After a full-term delivery, a mother notices a slight yellowing of her baby's skin. Test results come back which show high levels of bilirubin. The doctor orders phototherapy and the jaundice improves. What type of bilirubin was causing the symptoms and why was phototherapy effective? A. Unconjugated, chemical changes increasing the water solubility of bilirubin B. Unconjugated, chemical changes decreasing the water solubility of bilirubin C. Conjugated, chemical changes increasing the water solubility of bilirubin D. Conjugated, chemical changes decreasing the water solubility of bilirubin E. Conjugated, no change in solubility of bilirubin

Correct answer A: After birth there is an increase of RBC destruction and immature bilirubin conjugating system leading to high levels of nonconjugated bilirubin. During phototherapy bilirubin absorbs the light, changes chemically, and then becomes more water-soluble. A. CORRECT: See above B. INCORRECT: Phototherapy results in an increase of bilirubin's water solubility. C. INCORRECT: Elevated unconjugated bilirubin due to an immature conjugating system in babies D. INCORRECT: Elevated unconjugated bilirubin due to an immature conjugating system in babies, phototherapy results in an increase of bilirubin's water solubility E. INCORRECT: Elevated unconjugated bilirubin due to an immature conjugating system in babies, phototherapy results in an increase of bilirubin's water solubility

New parents bring their 1 month old to the ER. Baby has not stopped crying for 24 hours. Patient history shows baby was in the ER two weeks ago for an ear infection. Upon examination white spots on babies' tongues are observed. Laboratory testing reveals the patient has oral thrush. Given the patients history, genetic testing is ordered and shows baby has severe combined immunodeficiency disease (SCID), an X-linked disorder that neither parent presents with. By what mechanism is the babies immune system compromised? A)Decreased production of B-lymphocytes resulting in the lack of JAK3 production B) Erythropoietin receptor that is unable to bind SHP-1 causing sustained activation of JAK2 C) Premature dephosphorylation of STAT dimers D) Inactive B-lymphocytes fail to activate JAK3 due to lack production of T-lymphocytes E) Mutated docking regions preventing the attachment of STATs F) Over-activation of T-lymphocytes causing decreased activation of B-lymphocytes

Correct answer D - (pg 1899) - SCID results in the lack of formation of mature T-lymphocytes which do not activate B-lymphocytes. This results in inactive JAK3. A)Decreased production of B-lymphocytes resulting in the lack of JAK3 production -Incorrect; production of B-lymphocytes is not affected, they are not activated B) Erythropoietin receptor that is unable to bind SHP-1 causing sustained activation of JAK2 - Incorrect; this a different genetic mutation than SCID C) Premature dephosphorylation of STAT dimers - Incorrect; JAKs phosphorylates STATs; STATs must be phosphorylated to dimerize, SCID prevents activation of JAK before this can occur. D) Inactive B-lymphocytes fail to activate JAK3 due to lack production of T-lymphocytes - Correct E) Mutated docking regions preventing the attachment of STATs - Incorrect; SCID malfunction occurs before STATs process F) Over-activation of T-lymphocytes causing decreased activation of B-lymphocytes - Incorrect; SCID is a result of lack of T-lymphocytes

A 35-year-old patient went to the hospital this morning because he is not feeling well. The patient appeared pale and often reported weakness, malaise, easy fatigability, dyspnea on mild exertion, and his urine came out dark colored. After several tests the patient was diagnosed with paroxysmal nocturnal hemoglobinuria. This type of anemia can be classified according to its underlying mechanism as one of the following: A. Blood loss: Acute blood loss. B. Increased red cell destruction (hemolysis): Cardiac traumatic hemolysis. C. Decreased red cell production: Infections of red cell progenitors. D. Increased red cell destruction (hemolysis): Deficiency of phosphatidylinositol-linked-glycoproteins. E. Decreased red cell production: Deficiencies affecting DNA synthesis.

Correct answer D- Paroxysmal nocturnal hemoglobinuria is a type of anemia caused by an increased red cell destruction (hemolysis) because of a deficiency of phosphatidylinositol-linked-glycoproteins, a protein that protects them from the body's immune system. A. Incorrect- This type of anemia is caused by excessive blood loss because of a trauma. B. Incorrect- This type of anemia is caused by defective cardiac valves. C. Incorrect- This type of anemia is caused by virus infection of red cell progenitors. One example is Parvovirus B19 infection. D. Correct answer- See above. E. Incorrect- This type of anemia is caused by nutritional deficiencies. Deficiencies affecting DNA synthesis are caused by B12 and folate deficiencies.

A 21-year-old male from Northern Europe presents to the ER complaining of fatigue, lightheadedness, and appears jaundiced. Upon physical examination there appears to be tenderness in the right upper quadrant of his abdomen. He claims the lightheadedness comes and goes, but it has become more debilitating over the last few days. An ultrasound is ordered along with a CBC. Ultrasound shows the presence of gallstones and CBC indicates slightly lowered hemoglobin and hematocrit. These lab findings point to a diagnosis of Hereditary Spherocytosis. What is responsible for the morphology of red blood cells in patients with Hereditary Spherocytosis? A. Inability to convert oxidized glutathione to reduced glutathione, resulting in reduced protection against oxidant injury B. Missense mutation in the B-globin gene, replacing a glutamine residue with a valine residue C. Splicing mutation that destroys the normal RNA splice junction and prevents the production of normal β-globin mRNA D. A frameshift mutation that results in the failure to produce proteins, resulting in destabilization and the shedding of membrane fragments E. Somatic mutation inactivating PIGA, resulting in a cellular deficiency

Correct answer is D: Hereditary Spherocytosis is an inherited disorder that is caused by defects in the red cell membrane skeleton that leads to spherocytosis of the red cells. A frameshift mutation results in the mutated alleles not producing proteins which results in defects in assembling the membrane skeleton as a whole and destabilizing the plasma membrane. The loss of membrane compared to cytoplasm results in red blood cells taking on the smallest possible diameter, a sphere. A. Incorrect: this is the mechanism of a G6PD deficient deficiency that results in anemia because of the reduced capacity to protect against oxidant injury B. Incorrect: this is the mechanism behind Sickle cell disease; this mutation results in the sickle hemoglobin gene (HbS) that is responsible for Sickle Cell Anemia C. Incorrect: this is one of the 3 major mutations that can result in B-thalassemia, and in this case, β0-thalassemia D. Correct: see above E.Incorrect: this is the mechanism behind Paroxysmal Nocturnal Hemoglobinuria, that results from mutations in PIGA that cause membrane-associated deficiencies

The ER admits a 7 year old male with a suspected gastrointestinal obstruction. His patient history shows 6 admissions within the past year for repeated bacterial infections, 3 of which resulted in ICU care. His mother also reports he has frequent skin infections and diarrhea. Family history documents the mother as a carrier of the CYBB gene mutation affecting neutrophil function. What is the most probable defect in neutrophil function causing the boy's symptoms? A. The NADPH oxidase system is unable to form cytochrome B558 on the phagolysosome plasma membrane B. The myeloperoxidase (MPO) system of is unable to produce hypochlorous acid C. The NADPH oxidase system is unable to activate Rac-2 GTPase D. Defensins cannot produce the antimicrobial peptide cathelicidins E. the myeloperoxidase (MPO) system is unable to convert reactive nitrogen intermediates (RNIs)

Correct answer: A The boy has X91 disease, an inherited chronic granulomatous disease resulting from a mutation of the CYBB gene located on the X chromosome. This gene encodes gp91, a protein that combines with p22 to form the membrane bound cytochrome B558 in the NADPH oxidase system. Without both the membrane-bound cytochrome B558 and GTPase activity, neutrophils cannot produce the reactive oxygen intermediates (ROIs) to immobilize and kill bacteria/fungi. B - incorrect; the CYBB mutation would have no effect on the MPO system because MPO is another method of neutrophil phagocytosis, separate from the NADPH oxidase system. C - incorrect; the CYBB mutation affects the cytochrome portion of the NADPH oxidase complex, not GTPase portion. D - incorrect; while neutrophils utilize defensins, they are not a part of the NADPH oxidase system, and are unaffected by the CYBB mutation. E - incorrect; The MPO system creates hypochlorous acid, not RNIs.

A 30-year-old male suffers a knee injury that requires him to need surgery. In preparation for surgery, he undergoes testing, which includes; blood work, an EKG, chest x-ray, and a complete physical. After having his blood work performed, his PCP informs him that his blood work reveals anemia that is concerning. His PCP orders additional tests that reveal the man is heterozygous of β+ for β-thalassemia. The patient indicates that he has not noticed any changes in symptoms. He only reports mild fatigue from time-to-time, but attributes it to his busy work and his family schedule. What could be the cause of the patient's lab findings given his lack of symptoms? A. Elevated levels of HbF B. The hemoglobin ∝ and β- globin chains are in a 1:1 ratio C. Increased HbA/HbF ratio D. Increased concentration of HbC molecules in patient's RBC E. Deletion HPFH mutation, removing the entire 𝛿- and β-gene, leading to a decrease in HbF

Correct answer: A - This patient is confirmed to have a β-thalassemia. HbF is the predominant hemoglobin in the fetal period. HbF is eventually converted to HbA via the process of hemoglobin switching. Hemoglobin switching is not perfect, most individuals continue to produce small amounts of HbF during their life. Patients with hemoglobinopathies have less severe illness if their HbF levels are high. An elevated HbF level in this patient would explain his lack of symptoms. A. Correct - See above B. Incorrect- patient has β-thalassemia; therefore the ratio would not be 1:1 C. Incorrect - high levels of HbF are what lead to less severe illness; therefore a high ratio would not necessarily indicate high HbF level D. Incorrect - This molecule is seen with a patient that has sickle cell anemia E. Incorrect - If the patient were to have this mutation, they would produce lower levels of HbF

A 28-year-old female patient returns to the Emergency Department after being previously diagnosed with Hemolytic Uremic Syndrome. She now complains of moderate/severe chest pain and shortness of breath accompanied by generalized weakness. Following some laboratory analysis, the physician determines that the patient is now suffering from a condition in which her RBC's are being destroyed as they pass through the small vessels of her body. Considering the patient's medical history, physician's new findings, and presentation of new symptoms, what extrinsic hemolytic hemolysis might the patient be suffering from? A. Microangiopathic hemolytic anemia (MAHA) B. Thrombotic Thrombocytopenic purpura C. HELLP syndrome D. Hereditary Spherocytosis (HS) E. Hereditary Elliptocytosis (HE)

Correct answer: A Explanation: Hemolytic Uremic Syndrome is caused when the small vessels of the kidneys become damaged and/or inflamed. This damage leads the formation of clots within the vessels that can damage RBC's that are attempting to pass through the clotted area leading to the presence of schistocytes, or fragmented erythrocytes in a Peripheral Blood Smear analysis. This traumatic destruction of erythrocytes can compound and is indicative of a more devastating diagnosis in Microangiopathic hemolytic anemia. A. Correct; HUS and Thrombotic Thrombocytopenic purpura are two of the leading causes of MAHA; HUS prior to the visit is the give-away. B. Thrombotic Thrombocytopenic purpura would lead to MAHA, and the pre-diagnosis of HUS indicates that the condition is worsened. C. HELLP syndrome is indicated by elevated liver enzymes and low platelet count as well as hemolysis. No indication of this has been made in the question. D. Hereditary Spherocytosis is caused by heterogenous congenital abnormalities of erythrocyte skeleton proteins, not HUS E. Hereditary Elliptocytosis is caused by inherited mutations that affect membrane protein and cytoplasmic protein interactions, not HUS

A 25 year old female comes into the ER with complaints of fatigue, lethargy, headaches, genital pruritus, and thin, greyish-white discharge. The patient admits to being sexually active and refuses to use contraceptives. The nurse takes the patient's vitals and finds increased heart rate, stroke volume, cardiac output, along with decreased systemic vascular resistance. The doctor orders a CBC test and finds increased erythrocytes, decreased hematocrit, 40% increase in plasma volume, sodium levels are decreased by 4 mEq/L, and urea nitrogen is 50% decreased. A gram-stain of the discharge reveals clue cells that are KOH+. Which of the following is associated with the patient's deficiency? A. Increased nucleotide synthesis B. Increased large, immature RBCs C. Increased PMNs D. Increased Methyl-THF E. Increased Iron concentration

Correct answer: B Explanation: All the vitals and the CBC test allude to the patient being pregnant. During pregnancy there is an increased demand for folate. Folic acid is needed for the development in the neural tubes. The thin, white discharge suggests the patient also has an STI caused by Gardnerella vaginalis. A. Incorrect, because the patient has a folate deficiency, they are not able to produce purines and pyrimidines. B. Correct, due to having a folate deficiency the patient has megaloblastic anemia. Folate assists in the production of RBCs. C. Incorrect, The patient has bacterial vaginosis causing no increase in PMNs D. Incorrect, Methyl-THF is seen in the metabolic pathway of folic acid. With decreased folate, there will not be an increase in Methyl-THF E. Incorrect, due to the folate deficiency causing megaloblastic anemia, it is associated with an iron deficiency due to the malabsorption because of the megaloblastic changes in the intestinal tract.

A newborn male was admitted to the NICU upon birth due to apparent skeletal deformities and coarse facial features including a depressed nasal bridge and an elongated face. Months later the patient began exhibiting mental disabilities and clouded corneas. Laboratory testing revealed a decreased level of mannose 6-phosphate with elevated levels of lytic enzymes in the patient's plasma. It is determined that the physical deformities are a result of these lytic enzymes causing tissue damage. The patient also has an accumulation of undegraded molecules in his cells. Which of the following best describes this patient's condition? A. Tay-Sachs Disease B. I Cell Disease C. G6PD Deficiency D. Creutzfeldt-Jakob Disease E. PKU

Correct answer: B - A mannose 6-phosphate deficiency is characteristic of I (Inclusion) Cell disease. Mannose 6-phosphate binds and tags lysosomal proteins so that these enzymes make their way to lysosomes. Since this patient does not have mannose 6-phosphate, these lysosomal proteins aren't recognized by the lysosome and are released into the extracellular space, resulting in the high plasma lytic enzyme levels. Because the lysosomes are not receiving these lytic enzymes they cannot function normally. Thus, substances that normally get degraded instead begin to build-up causing the appearance of inclusions. A. Incorrect - Like I Cell disease, Tay-Sachs is also a lysosomal storage disease but is deficient in hexosaminidase A and causes a buildup of fatty substances. B. Correct - see above C. Incorrect - enzyme disorder that causes hemolytic anemia D. Incorrect - neurodegenerative disorder caused by prions E. Incorrect - PKU is caused by a buildup of phenylalanine in the blood and brain due to a defect in the gene encoding phenylalanine hydroxylase

Alexa, A 19-year-old female of Mediterranean decent seeks advice from her doctor after feeling abnormally fatigued. Upon physical examination, the doctor notices Alexa is more pale than usual. Previously, she had not sought medical attention because she had attributed her fatigue to working tirelessly to become Valedictorian in high school. Before high school, Alexa claims she was asymptomatic. Her doctor orders a CBC, hemoglobin electrophoresis, and molecular genetic testing. Lab results find that she has microcytic, hypochromatic RBCs due to decreased b-chain synthesis. What other lab result can be attributed to why Alexa was previously asymptomatic? A. Decreased HbF B. Elevated HbF C. Excess b-chains D. d0/b0- heterozygous E. b0/b0 homozygous

Correct answer: B - Alexa has b-thalassemia intermedia with mild symptoms of anemia. In b-thalassemia patients, b-globin synthesis is either reduced or is absent. However, if an individual is found to have elevated levels of Fetal Hemoglobin (HbF), oxygen is more likely to bind to HbF than HbA, reducing the effects of diminished b-chain synthesis in HbA on the individual. This is due to HbF containing the g-globin which has a greater affinity for oxygen than HbA. A. Incorrect- Decreased HbF in the case of anemia means there is a reduced oxygen in circulation, therefore symptoms would be worsened. As discussed in the correct answer, increased HbF can compensate for depleted oxygen levels. B. Correct: see above C. Incorrect- Because b-chain synthesis is diminished, they would not be in excess. D. Incorrect- d0/b0 heterozygous individuals have a deletion form of HPFH that do not produce enough HbF to compensate for diminished b-chain synthesis. E. Incorrect- b0/b0 homozygous individuals have a severe form of b-thalassemia and would not be asymptomatic for that length of time.

A 22-year-old male is treated with sulfonamides for a recent Urinary Tract Infection. A few days later he experiences fatigue, jaundice and dark urine and goes in for a follow up visit the next day. The doctor notices that he has a pale appearance and conducts a few lab tests. The results showed that his hematocrit is at 24% and his spleen is enlarged. The doctor then performs a crystal violet staining of the peripheral blood smear. What was found in this smear that indicates the diagnosis? A. Reticulocytes B. Heinz Bodies C. Chronic Hemolysis D. Phenacetin E. Young RBCs

Correct answer: B - Antibiotics along with pale skin, dark urine and low hematocrit indicates G6PD deficiency. Without G6PD, there is an increased susceptibility to free radical damage and oxidation of red blood cells. In this case, the oxidant drug sulfonamide causes hemoglobin to experience hemolysis resulting in the lower hematocrit and enlarged spleen.Denatured hemoglobin form Heinz Bodies that can be seen in crystal violet staining. A. Incorrect - Reticulocytes are not seen in the crystal violet staining. Instead, they cause patients with G6PD deficiency to have a higher enzyme level B. Correct - see above C. Incorrect - Although hemolysis is occurring, heinz bodies are what can be seen in lab tests as a result of the hemolysis. D. Incorrect - Phenacetin is an antibiotic that can cause hemolysis but cannot be seen in a crystal violet stain E. Incorrect - This is similar to Reticulocytes. Young RBCs can increase the enzyme level during acute episodes

A 30 year old female presents to her family physician with cramps, fever, nausea, and complaints of watery and bloody diarrhea. The physician notes that the patient also decided to self-medicate with leftover antibiotics right after she returned from goat yoga with her friends. After immediately being referred to the ER for dialysis, later lab tests show presence of oxidase negative, indole positive, lactose fermenting, Sorbitol negative organism in her stool. Which of the following best describes the bacteria's classification and structure? A. Gram positive bacteria, thick peptidoglycan layer with teichoic acid B. Gram negative bacteria, thin peptidoglycan layer with lipid A component C. Gram positive bacteria, thick peptidoglycan layer with LPS D. Gram negative bacteria, thin peptidoglycan layer with teichoic acid E. Gram negative bacteria, thick peptidoglycan layer with H antigen

Correct answer: B - Collective symptoms of cramps fever, nausea, and watery/bloody diarrhea combined with patient history of animal contact are all characteristic of enterobacteria which are gram negative bacteria. Additional clues hint specifically towards EHEC due to presence of an oxidase negative, indole positive, lactose fermenting, sorbitol negative organism in her stool. Answer choice B also aligns with characteristics of gram negative bacteria by itself, with a thin peptidoglycan layer containing a lipid A component that is present within the outer membrane's LPS. A. Incorrect: Gram positive bacteria do have a thick peptidoglycan layer with teichoic acid. However, a gram positive bacteria does not align with all lab diagnostics for EHEC, a gram negative organism. B. Correct: (see above) C. Incorrect: Gram positive bacteria do have a thick peptidoglycan layer, but LPS is only part of gram negative bacteria. Additionally, the lab diagnostics are indicative of EHEC, a gram negative bacteria D. Incorrect: Gram negative bacteria do have a thin peptidoglycan layer and also match the lab diagnostics for EHEC. However, their thin peptidoglycan layer does not contain teichoic acid. Teichoic acid is part of Gram positive bacteria's thick peptidoglycan layer. E. Incorrect: Though the laboratory diagnostics are indicative of EHEC, a gram negative bacterium with an H antigen on its motile flagella, gram negative bacteria have a thin (not thick) peptidoglycan layer.

An 8-year-old male was presented to his pediatrician with complaints of fatigue and shortness of breath. His pediatrician preformed a complete physical exam, including a CBC. His red blood cell count happened to be 5.7 x106/μL (normal RBC counts for children are 4.0 to 5.5 x106/μL of blood). It was later found that the child had a mutant erythropoietin (Epo) receptor that is unable to bind SHP-1. Erythropoietin is the hematopoietic cytokine that stimulates production of RBCs. What is true about SHP-1's role regarding RBC production? a) SHP-1 binds to and activates the Epo receptor b) SHP-1 binds to and deactivates the Epo receptor c) SHP-1 causes sustained activation of STAT 5 d) SHP-1 increases RBC production e) SHP-1 phosphorylates and activates JAK2

Correct answer: B - SHP-1, a tyrosine phosphatase, functions to dephosphorylate JAK2, thereby inactivating it. JAK2 is a non-receptor tyrosine kinase that transduces the signal from several cytokine receptors including the Epo receptor. JAK2 can activate multiple signal transduction pathways, including activation of several STAT proteins. These STAT proteins travel to the nucleus once activated and initiate gene transcription. If SHP-1 is unable to bind to the mutant Epo receptor, erythropoietin causes sustained activation of JAK2 and STAT 5 and, therefore, a higher-than-normal RBC count. SHP-1's role, out of the options given, regarding RBC production is binding to and deactivating the Epo receptor which negatively regulates RBC production. a) Incorrect: SHP-1 binds to and deactivates, not activates, the Epo receptor. b) Correct: see above c) Incorrect: SHP-1, a tyrosine phosphatase, functions to dephosphorylate JAK2, thereby inactivating it. JAK2 can activate multiple signal transduction pathways, including activation of several STAT proteins; therefore, if JAK2 is inactivated by SHP-1, STATs are also inactivated. d) Incorrect: SHP-1's role regarding RBC production is binding to and deactivating the Epo receptor which leads to the negative regulation of RBC production. In other words, SHP-1 leads to the deactivation and decrease of RBC production. e) Incorrect: SHP-1, a tyrosine phosphatase, functions to dephosphorylate JAK2, thereby inactivating it.

A 20-year-old female college student is exercising at the campus gym when she experiences difficulty breathing and palpitations. An employee, noticing her pale complexion and seemingly extraordinary exhaustion, escorts the student to the school health center. Blood work reveals reduced RBC mass. An inborn error in what type of metabolism could cause the student's blood disorder? A. amino acid B. carbohydrate C. porphyrin D. steroid E. purine/pyrimidine

Correct answer: B - inborn errors of glucose (carbohydrate) metabolism decrease erythrocyte survival by reducing RBC resistance to oxidative stress, which irreversibly damages structure, leading to hemolysis (hemolytic anemia) and removal from circulation. A. Incorrect - RBCs lack organelles; without organelles, specifically mitochondria, the cells cannot metabolize substrates other than glucose, including amino acids B. Correct - see above C. Incorrect - RBCs lack organelles; without organelles, specifically mitochondria, the cells cannot metabolize substrates other than glucose--heme (a porphyrin ring) cannot be synthesized D. Incorrect - RBCs lack organelles; without organelles, specifically mitochondria, the cells cannot metabolize substrates other than glucose, including steroids E. Incorrect - mature RBCs lack a nucleus and therefore do not have/need nucleic acid components

A 25 year old male presents to the clinic with complaints of a penile ulcer. The patient states this occurred 1 month after his vacation in São Paulo, Brazil. He further admits to having unprotected sexual intercourse multiple times during his time in Brazil. Lab results have narrowed down the causative agent to a species in the Enterobacteriaceae family. What is the shape of the causative agent and how will it present on a MacConkey agar ? A. rod, clear B. rod, pink/purple C. rod, green D. cocci, pink/purple E. cocci, cannot grow on MacConkey agar

Correct answer: B - rod [correct, as Klebsiella is part of the Enterobacteriaceae family], pink/purple [correct, as it is a lactose fermenter] A ) rod [correct, as Klebsiella is part of the Enterobacteriaceae family], clear [incorrect, as it is a lactose fermenter] B ) Correct - see above explanation C ) rod [correct, as Klebsiella is part of the Enterobacteriaceae family, a rod shaped family], green [incorrect, organisms will either be clear/pink+purple on MacConkey agar] D ) cocci [incorrect, as Klebsiella is part of the Enterobacteriaceae family, a rod shaped family], pink/purple [correct, as it is a lactose fermenter] E ) cocci [incorrect, as Klebsiella is part of the Enterobacteriaceae family, a rod shaped family], cannot grow on MacConkey agar [incorrect, as Klebsiella can grow on MacConkey agar]

A 22-year-old male patient presents to the Emergency Department with complaints of fatigue, weakness, chest pain, and headache. The patient explains that he has been getting much more exercise recently due to joining a church group basketball league. The patient also explains that he has been diagnosed with anemia. Which laboratory diagnostic test would the physician most likely order first, and what critical distinction might the results of the lab indicate to the physician? A. Peripheral Blood Smear; Increased RBC destruction or bleeding B. Reticulocyte Count; Primary failure of RBC production C. Erythrocyte Index; percentage of total number of RBC's D. Reticulocyte Count; Cause of anemia E. Peripheral Blood Smear; presence of hypoproliferative anemia

Correct answer: B Explanation: The patient has already indicated to the physician that he suffers from anemia, so the diagnosis has been predetermined. Anemia results in an elevated reticulocyte count due to the increase in production of erythropoietin which then stimulates the increased release of reticulocytes found present in the peripheral blood smear. A. This distinction is accomplished via a reticulocyte count, not from Peripheral Blood Smear. B. Correct; The second portion of answers A & B are the two critical distinctions indicated from a Reticulocyte Count. C. Erythrocyte indices tests would most likely not occur until after the PBS and Reticulocyte Count, and the percentage of total RBC's is accomplished via the reticulocyte count. D. This answer is too vague to be correct; answer B is a much better choice. E. Bone marrow examination is used to determine the presence of hypoproliferative anemia, not PBS

A 23 year old female presents to her primary care provider complaining of fatigue and decreased exercise tolerance. She states that she recently became vegetarian and reports a heavy menstrual cycle with an exacerbation of her symptoms. The PCP suspects Iron deficiency anemia and orders an iron study, which of the following lab results would confirm this diagnosis? Iron Concentration Total Iron Binding Capacity Transferrin Saturation Ferritin Saturation MCV A Decrease Increase Normal Decrease Macrocytic B Decrease Increase Normal Decrease Normocytic C Decrease Decrease Normal Normal Microcytic D Increase Increase Normal Decrease Microcytic E Increase Decrease Normal Normal Normocytic

Correct answer: B Iron deficiency anemia in premenopausal women is often related to increased iron loss with menstruation. Iron is acquired through the diet via two methods: heme (meat) and nonheme (vegetable) sources. Iron from heme sources is better absorbed making vegetarians more prone to iron deficiency anemia. A decrease in iron absorption leads to a decrease in serum iron concentration. This decrease in concentration also leads to a decrease in ferritin saturation since ferritin stores iron. During early stage iron deficiency anemia, MCV is often normal and later becomes microcytic, hence the importance of ordering an iron study. A. INCORRECT Iron deficiency anemia is normocytic or microcytic, never macrocytic. B. CORRECT. See above. C. INCORRECT A decrease in the iron concentration, decrease in TIBC, and normal ferritin saturation is indicative of chronic inflammatory disorder, not iron deficiency anemia. Iron deficiency anemia causes an increase in TIBC. D. INCORRECT Iron deficiency anemia is due to decreased iron concentration levels. E. INCORRECT Iron deficiency anemia is due to decreased iron concentration levels, decrease in iron concentration results in a decrease in ferritin levels due to the decreased iron storage demand.

A 22-year-old female patient visits the emergency department due to having a "pain crises." Her history shows that she suffers from sickle cell disease and is presenting today with severe body pains. Upon examination of her peripheral blood smear, irreversibly sickled cells can be visualized. Her blood work is indicative of hemolytic anemia, and reticulocytosis. This patient has increased susceptibility of which of the following? A. Falciparum malaria B. Haemophilus influenzae C. Acute Osteomyelitis D. Hypobilirubinemia E. Decreased occlusion of vascular beds

Correct answer: B Patients that suffer from sickle cell disease are more susceptible to infection with encapsulated organisms. Due to the altered splenic function being impaired by poor blood flow in children or being completely absent in adults due to splenic infarction. A. Incorrect: High prevalence of sickle cell trait in African populations stems from its protective effects against falciparum malaria. Patients with sickle trait are less likely to have severe disease or to die from malaria. B. Correct: see above C. Incorrect: In children painful bone crises are extremely common and often difficult to distinguish from acute osteomyelitis. D. Incorrect: Major pathologic manifestations are moderately severe hemolytic anemia associated with reticulocytosis, and hyperbilirubinemia E. Incorrect: Sickle cell disease causes microvascular obstruction and ischemic tissue damage

A 35-year old male was recently diagnosed with a rare hemolytic disease, Paroxysmal Nocturnal Hemoglobinuria (PNH). A unique feature of this disease is the tendency for RBCs to lyse at night/during sleep. What biological feature is responsible for this event occurring? A) Deletion of a single α-globin gene B) Slight decrease in blood pH C) Splicing mutation D) Mycoplasma pneumoniae infection E) G6PD deficiency

Correct answer: B- Slight decrease in blood pH during sleep. This decrease in pH increases the activity of complement, allowing antibodies to recognize and bind red blood cell antigens. A. Incorrect- cause of silent carrier state in α-Thalassemia B. Correct- see above C. Incorrect- splicing mutations are one of the major causative reasons of ß-Thalassemia D. Incorrect- Mycoplasma pneumoniae is typically associated with cold agglutinin type immunohemolytic anemia E. Incorrect- deficiencies in G6PD result in accumulated oxidative damage in G6PD related hemolytic anemia

A 44-year-old female presents to her gynecologist with malaise, fatigue, shortness of breath, and severe pain in the pelvic area. Although her menstrual cycles are regular, and she suffers from dysmenorrhea and menorrhagia. Lab tests primarily reveal a hemoglobin level of 10 g/dL (normal: 12-16 g/dL female), a hematocrit level of 34% (normal: 37-47% female), and an MVC level of 73 fL (normal: 80-95 fL). An ultrasound reveals a benign mass in her uterus that is exerting pressure on her urethra and bladder, leading to difficulty and pain in emptying urine, thus causing frequent UTIs. Before resorting to surgical options to remove the fibroid, the gynecologist wants to try other treatment methods to help the patient's symptoms. Which of the following treatments is best for this patient? A. Blood transfusion B. A diet rich in meat and dark, leafy greens C. Treatment with hydroxyurea D. A diet which avoids consumption of fava beans E. Treatment with antibiotics

Correct answer: B. A diet rich in meat and dark, leafy greens The patient has a form of anemia caused by blood loss, specifically chronic blood loss. She presents with symptoms of anemia/ iron deficiency anemia such as fatigue, shortness of breath, malaise, and the chronic blood loss comes from gynecologic disturbances such as menorrhagia (heavy menstrual bleeding). Her lab results show decreased levels of hemoglobin, hematocrit, and MVC. Chronic blood loss induces anemia only when the rate of loss exceeds the regenerative capacity of the marrow or when iron reserves are depleted and iron deficiency anemia appears. Of all answer choices for possible treatment options for this patient based on her symptoms and lab test results, having a diet rich in meat and dark, leafy greens is best as these foods are rich in iron, which is crucial for red blood cell production. The iron is found in hemoglobin, which is essential for transferring oxygen from blood to tissues. A. Blood transfusion - incorrect because blood transfusions are used by various hemolytic anemias primarily. The transfusions have different purposes and consequences in each disorder, however none of that applies to this case since it is about anemia caused by chronic blood loss. B. A diet rich in meat and dark, leafy greens - correct see explanation above C. Treatment with hydroxyurea - incorrect because hydroxyurea is an inhibitor of DNA synthesis which is used to treat sickle cell disease by causing an increase in red cell HbF levels and an anti-inflammatory effect due to an inhibition of leukocyte production. D. A diet which avoids consumption of fava beans - incorrect because this treatment is specific to the G6PD deficiency. The patient in this case does not show certain key characteristics pertaining to this disorder. E. Treatment with antibiotics - incorrect because the main symptoms are not pertaining to the patient's UTIs, rather they align with gynecological disturbances and anemia. The correct answer for this question would be some form of treatment that would alleviate the patient's anemic symptoms.

An 11 month old child presents to the clinic after the rapid development of watery diarrhea accompanied by fever and vomiting starting within hours after arriving home from daycare yesterday. The parents deny any blood in the patient's stool and also say they have not fed him any undercooked meat. It is determined that the illness was spread via fecal oral from another child attending daycare. After lab tests confirm the doctors diagnosis, it is known that this specific strain possesses a cluster of virulence genes located on a locus of enterocyte effacement (LEE). What is the likely mechanism this bacteria uses to initiate infection causing watery diarrhea? A. Shiga toxins (Stx1 and Stx2) B. Attachment to epithelial cells leading to effacement of microvilli (A/E histopathology) C. Heat stable toxin (STa or STb) causing an increase in cAMP D. Invasion and destruction of colonic epithelium E. Heat labile toxin (LT-I or LT-II)

Correct answer: B. This is a description of Enteropathogenic E. coli. This can be determined based on the fact that the patient is an infant, transmission was fecal oral person to person, rapid onset of symptoms and the absence of blood in the stool. Virulence factors on a LEE pathogenicity island rules out ETEC as a diagnosis. A. Incorrect - Shiga toxin (associated with Shigella or EHEC) would likely cause bloody diarrhea and is spread via undercooked meat or unpasteurized milk or juice. B. Correct! C. Incorrect - C and E are both incorrect because ETEC toxins are spread via consumption of contaminated food/water. It cannot be spread person to person. (C is also incorrect because heat stable toxin causes an increase in cGMP, not cAMP). D. Incorrect - this is not Enteroinvasive E. coli which would present with bloody diarrhea. E. Incorrect - See explanation for C

A 32 year old female presents to the Emergency Room with a 3 day history of explosive watery diarrhea, with sudden onset on her flight back from Mexico. The physician sends a blood sample to the lab, and 24 hrs later the culture results with growth on MacConkey Agar with lactose fermentation. The oxidase test is negative, and the catalase test is positive. The physician identifies the organism as Escherichia coli. Which virulence factor(s) of the causative organism is responsible for the symptoms exhibited by the 32 year old female? A. P pili B. Invasive plasmid antigen C. LT-1, STa D. Stx1, Stx2 E. Capsule

Correct answer: C Lab results have isolated the organism as E.coli, and given the patient's recent travel history to Mexico and sudden onset of watery diarrhea the strain can be narrowed down to ETEC (Enterotoxigenic E.coli). The exotoxins associated with ETEC are heat labile LT-1 and heat stable STa. A. Incorrect: adhesin associated with UTIs (uropathogens) B. Incorrect: associated with Enteroinvasive E.coli, symptoms progress from watery to bloody diarrhea C. Correct - see above D. Incorrect: shiga toxins associated with Enterohemorrhagic E.coli, symptoms progress from watery diarrhea to gross bloody diarrhea E. Incorrect: capsule allows Enterobacteriaceae to be protected from phagocytosis and is not the cause of increased cAMP/cGMP associated with the patient's watery diarrhea

A 30-year-old male patient was brought into the ED by his friends after sweating profusely, having a flushed face appearance, and eventually blacking out at a Superbowl party. Lab results revealed tachycardia (120 bpm), high blood ethanol levels (405 mg/dL), and a low MCV (75 fL/cell). A peripheral blood smear revealed small, hypochromic erythrocytes. The physician would most likely diagnose the patient with what type of microcytic anemia and its effect? Iron deficiency; lack of heme synthesis results from the absence of iron incorporating into the porphyrin ring Thalassemia minor; globin synthesis disruption Acquired Sideroblastic anemia; inhibition of the heme synthetic pathway enzymes Congenital Sideroblastic anemia; possible response to pyridoxine and therefore inhibition of the heme synthetic pathway enzymes Lead poisoning;blocks iron incorporating into heme

Correct answer: C - Acquired Sideroblastic anemia's most common cause is alcohol abuse. With the lab results indicated, there is a high level of ethanol in the blood. Ethanol will inhibit most of the enzymes in the heme synthesis pathway. A. Although iron deficiency is the most common cause of anemia, the specific results given pointed to more than a classic iron deficiency.Iron levels would need to be taken into consideration regardless of the MCV level. B. Thalassemia is a result of hemoglobinopathies (inherited blood disorders) and none of the signs and symptoms given indicated this. D. Congenital is from birth and from the scenario given this indicates it was probably an acquired anemia due to alcohol abuse. E. Lead poisoning wouldn't be thought of based on this scenario but had it been a career listed that could be exposed to lead this would be a possible answer.

A 12 year-old female presents to the emergency department with fatigue and pallor. Upon H/P exam she reports that she began menstruating 5 months ago and doesn't eat any meat, and iron deficiency is suspected. A routine reticulocyte count is performed and is found to be particularly low. Which test would further confirm iron-deficient anemia based on the size of erythrocytes? A. Peripheral blood smear B. Peripheral blood iron indices C. Mean corpuscular volume (MCV) D. Bone marrow aspirate E. Nutritional exam

Correct answer: C - The MCV is helpful in Dx of anemia when reticulocyte count is low. A direct measurement of the volume of erythrocytes can further characterize anemias as microcytic, macrocytic, or normocytic. (Note*- iron deficient anemia may need further testing and cannot be confirmed solely from MCV since it takes on normocytic and microcytic stages) A. A peripheral blood smear would reveal the morphology of erythrocytes and other blood cells but would not help characterize anemia based on the size of the erythrocytes B. Peripheral blood iron indices are a group of several specific tests that may definitively confirm a Dx for iron deficiency. However, the question asks which test would best identify the anemia based on size of erythrocytes, thus making this option incorrect. D. Bone marrow aspirate or examination would confirm or refute the Dx of iron deficient anemia based on the presence of iron in the bone marrow, not based on the size of erythrocytes. E. A nutritional exam would only provide information regarding the patient's nutritional status and would not provide any information regarding the size of her erythrocytes.

After Frodo noticed that his 2 year-old son, Samwise was constantly fatigued, he brought him to their pediatrician. The physician noticed that the toddler had a pallor complexion and after the physician examination, she suspected that the toddler had splenomegaly. She decided to draw blood and ordered a Complete Blood Count with Differential. Later that evening, the Pathologist called her and said that he noticed that the erythrocytes were elliptical in shape. What two proteins are most commonly associated with the shape of the erythrocytes seen? A. DAF & MIRL B. DAF & CD59 C. Spectrin & Protein 4.1 D. Spectrin & Protein 4.2 E. Spectrin & Ankyrin

Correct answer: C - This patient has Hereditary elliptocytosis which is caused by mutations affecting how the membrane proteins interact with the underlying cytoplasmic proteins. The most common abnormalities affect the interactions with spectrin and protein 4.1, which causes the RBCs to assume an elliptical shape. Table 47-7 also features the correct answer. A. Incorrect - These two proteins are not associated with Hereditary elliptocytosis. They are membrane proteins that are located on normal erythrocytes. DAF is a delayed accelerating factor protein and MIRL is a membrane inhibitor of reactive lysis protein. B. Incorrect - These two proteins are not associated with Hereditary elliptocytosis. They are membrane proteins that are located on normal erythrocytes. DAF is a delayed accelerating factor protein and CD59 is a membrane inhibitor of reactive lysis protein. C. Correct - See Above. D. Incorrect - Spectrin is associated with Hereditary elliptocytosis, but protein 4.2 is not. However, abnormalities in these two proteins are both associated with Hereditary spherocytosis (HS). E. Incorrect - Spectrin is associated with Hereditary elliptocytosis, but ankyrin is not. This answer would be correct if the question was asking for which two proteins were most commonly associated with HS. Most patients with HS have dominantly inherited mutations in spectrin or ankyrin.

A 30 year old female patient presents to her primary care physician with concerns of her constant watery and bloody diarrhea. Her diet has been normal for her with the exception of eating a rotisserie chicken that she picked up from the grocery store on her way home from work approximately two days ago. A culture has been done and growth on MacConkey agar shows the organism is a non-fermenter and resistant to bile salts. What is the most likely causative organism for the patient's symptoms? A. E. coli B. Staphylococcus aureus C. Bacillus cereus D. Shigella dysenteriae E. Vibrio parahaemolyticus

Correct answer: D. The best answer to this question is Shigella dysenteriae. Broad clues are given in the question with the description of the stool and an out of the ordinary food decision. However, the growth on MacConkey agar and the results that it is a non-fermenter and resistant to bile salts drives home the conclusion that it must be Shigella dysenteriae. A. Incorrect- E. coli grows on MacConkey agar, however, it is a lactose fermenter B. Incorrect- S. aureus is a gram positive cocci and would not be grown on MacConkey agar. C. Incorrect- Bacillus cereus is a gram positive rod that forms endospores and would not be grown on MacConkey agar. D. Correct- see explanation above. E. Incorrect- Vibrio parahaemolyticus is a gram negative curved rod. The symptoms for it are caused by improperly cooked seafood and in this question we are looking at improperly prepared chicken.

A 35 y/o male presents to the ED after feeling extremely weak and dizzy for the past 3 days. The patient states that his dizziness has become progressively worse, and has started to feel short of breath. The patient mentions to the physician that he is currently being treated for Non-Hodgkin's lymphoma and is receiving chemotherapy once every two weeks. A CBC test was completed with normal findings other than a low RBC count. Further testing was done showing elevated hepcidin levels. What treatment would be most appropriate to start with for this patient? A. High-dose oral and parenteral cobalamin B. Ferrous sulfate C. Correction of nutritional deficiencies with combined Iron supplementation D. Erythroid stimulating agents (ESA) E. Stem cell transplantation

Correct answer: C - This patient has the signs and symptoms of Normocytic anemia. A big clue in the question is that the patient is currently being treated for cancer. Another factor is that all blood work came back normal, other than the decreased RBC number on a CBC. Patients with this anemia of chronic infections will have elevated hepcidin levels that causes the reduction in intestinal iron absorption and iron mobilization from macrophages. You have to be careful with treating cancer patients, and it is recommended that you first correct the nutritional deficiencies and give Iron supplementation. A. Incorrect- This would be a treatment used for those with Megaloblastic anemia. B. Incorrect- This is used to help treat those with Microcytic anemias. They will be given ferrous sulfate 2-3 times daily. C. Correct- see above explanation D. Incorrect- This is a treatment used in Normocytic anemias and has been seen to reduce transfusion needs in many of these patients. However, the patient in this question is being treated for cancer. (pg 508) states that EPAs should be avoided in cancer patients if being treated, and our example patient is being treated with chemotherapy at this time. E. Incorrect- Stem cell transplantation has been seen as a therapeutic treatment for those with Aplastic anemia. This focuses on replacing the defective HSC by stem cell transplantation or controlling an overactive immune response.

Which of the following enzymes is correctly matched to the heme synthesis disorder most likely occurring in a patient who presents to the clinic with symptoms consisting of anxiety, light sensitivity, and increased facial hair? a) Protoporphyrinogen oxidase, acute intermittent porphyria b) Delta-ALA dehydratase, Lead poisoning c) Uroporphyrinogen decarboxylase, porphyria cutanea tarda d) Uroporphyrinogen decarboxylase, congenital erythropoietic porphyria e) Cytochrome b5 reductase, methemoglobinemia

Correct answer: C - the deficient enzyme and Porphyria are correctly matched; with cutanea tarda, dermatological photophobia is common, and Porphyrias are classically associated with Neuropsychiatric symptoms, and "werewolf" like symptoms, with increased scarring and facial hair. a) Incorrect—while Protoporphyrinogen oxidase is a heme synthesis enzyme that can be deficient leading to a Porphyria, it is incorrectly matched with Acute intermittent porphyria. It should be matched with Variegate Porphyria. b) Incorrect—inactivation of this enzyme is associated with lead poisoning, which causes delta-ALA and Protophyrin IX to accumulate and heme production decreases. However, lead poisoning would not cause neurological symptoms, nor photophobia or increased facial hair, as these are more indicative of a Porphyria. c) Correct: see above d) Incorrect—While Uroporphyrinogen decarboxylase is a heme synthesis enzyme that can be deficient leading to a Porphyria, it is incorrectly matched with Congenital erythropoietic porphyria. It would be matched with Porphyria cutanea tarda. e) Incorrect—Cytochrome b5 reductase is associated with congenital methemoglobinemia. However, these are not associated with the heme synthesis pathway. Congenital methemoglobinemia is the presence of excess methemoglobin found in people with cytochrome b5 reductase deficiency. These individuals also do not experience the symptoms listed in the question, and instead generally only experience cyanotic tendencies with few other clinical symptoms.

A 32 year old female presents to her physician complaining of chronic itching, abdominal pain, nausea and loss of appetite. She also states that she has noticed red marks developing under the surface of the skin around the area where the itching has been occurring. She says that she spends a lot of time outside camping and hiking with her dog. Further examination reveals that she is suffering from an infection caused by the parasite Necator americanus. Which type of blood cell will most likely be active at the site of infection? A. Neutrophil B. Basophil C. Eosinophil D. Erythrocyte E. Thrombocyte

Correct answer: C- Eosinophils are a form of granulocytic white blood cells. One of their main responsibilities is to fight against parasites. Their granules are filled with hydrolytic enzymes and proteins that are toxic towards parasitic worms. A. Incorrect: Neutrophils are phagocytic white blood cells. They can be found at the site of most infections but are not limited to parasitic infections. B. Incorrect: Basophils are granulocytic white blood that are most active at the site of allergic reactions C. Correct: Explanation above D. Incorrect: Erythrocytes can be found throughout the majority of the body. Their main responsibility is to transport oxygen to the various tissues. E. Incorrect: Thrombocytes, also commonly called platelets, are found at clotting sites

A 19-year-old female presents to her primary care physician complaining of easy bruising on her arms and frequent nosebleeds. Her physician orders a CBC blood test, which shows a low platelet count. Which of the following platelet secretions is responsible for reducing blood flow to areas of blood vessel injury. A. Thromboxane A2 B. λ granules C. Serotonin D. Heparin E. α granules

Correct answer: C- Serotonin is released in response to platelet adhesion. It functions as a vasoconstrictor, resulting in decreased blood flow to areas of damaged blood vessels. A. Incorrect- Thromboxane A2 functions as a signaling molecule in platelets B. Incorrect- λ granules are responsible for clot resorption C. Correct- see above D. Incorrect- Heparin is an anticoagulant and is not secreted by platelets E.Incorrect- α granules are important in blood vessel repair, coagulation and platelet aggregation

A 2-day old newborn develops neonatal jaundice, a condition defined by a yellow tint to the skin and eyes due to elevated erythrocyte destruction leading to high levels of bilirubin. Total bilirubin is the addition of direct and indirect bilirubin measured in the blood. Which of the following is not true regarding direct and indirect bilirubin? A. Conjugation of indirect bilirubin occurs in the liver. B. Indirect bilirubin is bound to albumin in the blood. C. Direct bilirubin is bound to albumin in the blood. D. Indirect bilirubin is more likely to be found deposited in lipid environments. E. Conjugation of bilirubin depends on UDP-glucuronate transferase.

Correct answer: C. After conjugation in the liver, direct bilirubin becomes water-soluble and no longer requires albumin for transport in the blood plasma. Direct bilirubin is then actively transported from the liver into the bile for excretion. All other answer choices are accurate descriptions of direct or indirect bilirubin. A. Incorrect - Conjugation does take place in the liver. B. Incorrect - Indirect (unconjugated) bilirubin is fat soluble and therefore requires help from albumin to move through the blood to the liver. C. Correct D. Incorrect - Indirect bilirubin is unconjugated and fat soluble, so it is more likely than direct (conjugated) bilirubin to be found deposited in fatty environments. E. Incorrect - Conjugation of bilirubin is dependent on the enzyme UDP-glucuronate transferase for the substitution reaction of bilirubin onto UDP-glucuronate.

Mark, A 44-year-old avid runner complained to his wife on their 20th wedding anniversary about an unusual bone pain and weight loss. Given his interest in exercise coupled with a keen interest in ketogenic diet. They both instinctually attributed these symptoms to possibly be induced by his diet regimen and exercising. However, three days later the pain progressively continued to worsen with developing fever, nosebleeds, and swollen lymph nodes. Upon visiting the ER, the attending physician decided to run a CBC blood test and conduct a bone marrow biopsy in order to make a differential diagnosis. The test results revealed that Mark has a 90% bone cellularity index (Hypercellular bone marrow) coupled with an excessive buildup of immature white blood cells. Which of the following is a plausible diagnosis the physician might be most concerned with? A. Aplastic anemia B. Subacute sclerosing panencephalitis C. Osteomalacia D. Acute myelogenous leukemia E. Hodgkin's lymphoma

Correct answer: D - Bone marrow cellularity serves as an important indicator in evaluating bone marrow function with its compositional ratio of hemopoietic cells to adipocytes changing as we age. In patients with Acute myelogenous leukemia, a tumor originating from hemopoietic cells triggers an increase in bone cellularity that causes a rise in the amount of myeloblast. A. Incorrect- Aplastic anemia represents a case of hypocellular bone marrow. In which case the bone cellularity index would be significantly decreased. B. Incorrect- SSPE is a progressive brain inflammation caused by a hypermutated strain of measles virus C. Incorrect- Osteomalacia results from an inadequate supplementation of vitamin D D. Correct- see above E. Incorrect- Although sharing similar symptoms, Hodgkin's lymphoma is a cancer of the lymphatic system that develops from abnormal B-cells

Glycoproteins are key structural components of the Extracellular matrix. Formation of Glycosaminoglycans (GAGs) requires that a sugar undergo conversion to an amino-sugar and subsequent acetylation to form GAGs such as Hyaluronic Acid. Which of the following is responsible for the addition of an amino group onto Fructose 6-Phosphate to form Glucosamine 6-Phosphate? a) Aspartate b) Serine c) Acetyl Co-A d) Glutamine e) Ammonium

Correct answer: D - Glutamine has its amino group transferred to Fructose 6-Phosphate and is released as Glutamate. The whole process starts with glucose (the starting sugar) which is converted to Glucose 6-Phosphate. Then conversion of Fructose 6-Phosphate to Glucosamine 6-Phosphate takes place upon receiving the amino group from Glutamine. Glucosamine 6-P is then the substrate for the acetylation reaction with Acetyl Co-A to form GAGs such as Hyaluronic Acid. a) Incorrect - Aspartate is an N-containing group and plays a role in the Urea cycle, however it was not mentioned in the chapter and does not play a role in donating an amino group to Fructose6-P in the formation of glycoproteins. b) Incorrect - Serine is the distractor answer as it was mentioned in the chapter NOT as a donator of an amino group to Fructose6-P, but as one of the three common residues on a protein to which a carbohydrate would covalently bind to form a glycoprotein. c) Incorrect - Acetyl Co-A does not donate an amino group to Fructose6-P. It is involved only in the acetylation of Glucosamine 6-Phosphate into N-Acetylglucosamine-6-Phosphate, which is the acetylation step that occurs after the addition of the amino group to the sugar d) Correct: see above e) Incorrect - Ammonium is an N-containing compound; however, it was not mentioned in chapter 27 and is not involved in the formation of the sugar precursors for glycoprotein synthesis

A 24-year-old male presented to his primary care physician with lower neck pain, migraines, and cutaneous nodules on his genitalia. After further examination, the physician diagnosed the patient with primary syphilis. After treating the patient for two weeks with Benzathine penicillin, the nodules turned into painless granulomatous lesions. After further discussing the history of the patient, the physician had learned that the patient recently traveled to Australia on a mission trip 3 months ago. What bacterial infection does this patient have? A. Klebsiella pneumoniae B. Citrobacter koseri C. Klebsiella ozaenae D. Klebsiella granulomatis E. Proteus mirabilis

Correct answer: D - Klebsiella granulomatis. Klebsiella granulomatis is rare in the U.S. but endemic in parts of Papua New Guinea, the Caribbean, South America, India, southern Africa, Vietnam, and Australia. It can be transmitted after repeated exposure through sexual intercourse or nonsexual trauma to the genitalia. Incubation of weeks to months, subcutaneous nodules appear on the genitalia. The nodules break down into painless granulomatous lesions that extend and coalesce into ulcers resembling syphilitic lesions. A. Incorrect - Causes community acquired or hospital acquired lobar pneumonia. Involves necrotic destruction of alveolar spaces, formation of cavities, and production of blood tinged sputum (Page 262). B. Incorrect - Causes meningitis and brain abscesses in neonates (page 263). C. Incorrect - Cause of chronic atrophic rhinitis (page 262). D. Correct- see above. E. Incorrect - Primarily produces infections of the urinary tract or cystitis. Cause renal (kidney) stones (page. 262-263).

Ahmad, 33-year-old male, presents to the doctor with hemolytic anemia and yellowing of the sclera of his eyes. Eventually the doctor determined that the likely cause of his symptoms was an X-linked genetic defect in the gene that codes for glucose 6-P dehydrogenase. If the diagnosis is accurate, what is the pathway in which Glucose-6-Phosphate dehydrogenase is the rate limiting enzyme and what change in activity would be associated with this genetic defect? a) Pentose phosphate pathway; Increased NAPDH levels and decreased superoxide synthesis b) TCA cycle; decreased NADPH and increased NADH c) Pentose phosphate pathway; Decreased NADPH levels and Increased superoxide synthesis d) Pentose phosphate pathway; Decreased NADPH levels and decreased Cytochrome P450 monooxygenase e) Not enough information given

Correct answer: D - Pentose phosphate pathway; Decreased NADPH levels and decreased Cytochrome P450 monooxygenase. NADPH is required for Cyt-c p450 production. a) Incorrect - g6pd is required for NADPH synthesis in the PPP. NADPH functions in the creation of superoxides. b) Incorrect - TCA cycle has little to do with g6pd. c) Incorrect - conflicting, NADPH is required in superoxide synthesis. d) Correct: see above e) Incorrect: enough information is given. NADPH is required for Cyt-c p450 production.

A 53-year-old male is seen in his cardiologist's office for follow-up regarding history of atrial fibrillation and flutter. He had received 2 infusions of procainamide 2 weeks prior to this office visit. Upon general appearance, the patient appears pale and fatigued. During the physical, the cardiologist noted bilateral conjunctival icterus. Bloodwork revealed decreased RBC count, elevated reticulocyte count and decreased hematocrit. Which of the following describes the mechanism of hemolysis for this patient? A. IgM antibodies are produced and directed against RBC antigen I B. Drug binds to erythrocytes and acts as a hapten; antibodies are produced and directed against the drug C. IgG antibodies are produced and directed against RBC antigen i. D. True anti erythrocyte antibodies are produced and directed against RBC antigens including Rh. E. An antibody-drug complex forms and binds to the erythrocyte membrane to active complement

Correct answer: D - Procainamide is one of the drugs that can cause drug-induced autoimmune hemolytic anemia (AIHA). His symptoms - paleness, fatigue and bilateral conjunctival icterus (yellowing of the eyes) - describe jaundice, which is a general sign of anemia. Drug-induced hemolysis is mediated by warm antibodies (IgG). There are 3 main mechanisms by which drugs can induce AIHA: hapten mediated, immune complex mediated and true anti-RBC antibody production. The mechanism of procainamide-induced hemolysis falls under true anti-RBC antibody production, which is what answer choice D describes: true anti erythrocyte antibodies are produced and directed against RBC antigens including Rh. A. INCORRECT -This is the mechanism for IgM-mediated (cold) AIHA. Warm antibodies (IgG) mediate drug-induced hemolysis. B. INCORRECT - This is the mechanism for penicillin- induced AIHA. Penicillin still falls under the category of drug-induced hemolysis, but for procainamide, antibodies are against the antigens of the RBC itself, not against the drug. C. INCORRECT - This is describing the mechanism for cold AIHA which is only mediated by cold antibody, IgM, not IgG. D. CORRECT - This describes the mechanism by which procainamide induced AIHA - by the production of true anti-RBC antibodies. E. INCORRECT - This is the mechanism of quinidine, quinine and rifampin - induced AIHA. Procainamide does not form an immune complex with the antibodies it produces, nor does it bind to the erythrocyte membrane.

A 27-year-old female has a blood smear and a complete blood count (CBC) preformed as a part of her routine physical. The patient's vitals are also recorded: BP 120/90, HR 90, and Temperature 98.7°F. The results reveal that the patient has a hemoglobin mutation. This mutation is located at the same position as the HbS hemoglobin mutation. The mutation the patient is diagnosed with is characterized by an amino acid change from Glu to Lys. The patient is clinically unaffected by this hemoglobin mutation. What effect is the hemoglobin mutation having on the patient's hemoglobin concentration within the cells? a) Decrease in water loss from the cell due to a suppression of the potassium transporter resulting in a decrease in concentration of hemoglobin in the cells b) There is no change in hemoglobin levels in the cells because the mutation does not clinically affect the patient c) There is a decrease in solubility of hemoglobin d) Increase in water loss from the cell due to an activation of potassium transporter resulting in an increase in concentration of hemoglobin in the cells e) The patient has sickle cell anemia

Correct answer: D - The hemoglobin mutation HbC has two effects and one of which is: promotes water loss from the cell due to an activation of potassium transporter that results in an increase in hemoglobin in the cell. a) Incorrect: There is an increase in water loss, the potassium transporter is activated, there is an increase in concentration of hemoglobin in the cells. b) Incorrect: There is an increase concentration of hemoglobin the cell. There is no clinical significance because this patient's HbC mutation is heterozygous NOT because there is no change in hemoglobin concentrations. c) Incorrect: The question is not asking about hemoglobin solubility. There is a decrease in solubility of hemoglobin in homozygote individuals. This patient is heterozygous. d) Correct: see above e) Incorrect: The patient does not have sickle cell anemia (HbS). The patient has hemoglobin mutation HbC.

A 4-year-old child presents to her primary care physician with nausea, vomiting, and diarrhea. Her mother claims she has not seen any blood in the diarrhea. She states that the symptoms started 12 hours ago, after her daughter attended an outdoor birthday party where the children were served chicken fingers. The mother also informs the physician that her daughter is not the only child who attended the party that is now feeling ill. When grown on MacConkey agar, this organism is seen to be lactose non-fermenting. Which of the following is NOT a characteristic of the causative organism? A. Replicates in endocytic vacuoles B. Attaches to the mucosa of the small intestine and invades M cells C. Stimulates cAMP to increase fluid secretion D. Associated with chronic diarrhea and growth retardation in children E. Genes encoded on pathogenicity islands

Correct answer: D - The organism causing gastroenteritis in this patient is a Salmonella species (lactose non fermenting). The child ate contaminated poultry at an outdoor social gathering, which has been noted to be one of the most common methods of exposure to Salmonella in her age group. Salmonella is not associated with chronic diarrhea and growth retardation in children, as this is a characteristic of EAEC (lactose fermenting), which would cause watery secretory diarrhea accompanied by fever, nausea, vomiting, and abdominal pain. A. Incorrect - this is a characteristic of Salmonella; after invading M cells, Salmonella remain in endocytic vacuoles to replicate B. Incorrect - this is a characteristic of Salmonella; Salmonella attach to the mucosa of the SI and invade M cells in Peyer patches and enterocytes C. Incorrect - this is a characteristic of Salmonella; Salmonella activates the inflammatory response of the host cell, which releases prostaglandins, stimulates cAMP, and active fluid secretion D. Correct - See above E. Incorrect - this is a characteristic of Salmonella; regulation of attachment, engulfment, and replication is controlled by two large clusters of genes: pathogenicity islands I and II

A 3-yr old male presents to the ER with fever, chest pain, difficulty breathing, and bluish tinge on fingers. It is determined the patient is experiencing acute chest syndrome and needs an emergency blood transfusion. A blood culture is performed and high levels of Salmonella are found in their system. What would this cause in the patient? A. Aplastic Crisis B. Hydroxyurea C. Heinz bodies D. Osteomyelitis E. Vitamin B12 Deficiency

Correct answer: D - patients with sickle cell disease are particularly prone to osteomyelitis from Salmonella. It is revealed that the patient has sickle cell disease because of acute chest syndrome. A. Aplastic Crisis - it can happen to a person with sickle cell disease but it is more associated with Parvovirus B19 B. Hydroxyurea - this is used as a treatment for sickle cell disease, not associated with Salmonella C. Heinz Bodies - this is associated with G6PD and not sickle cell disease D. Osteomyelitis E. Cobalamin Deficiency - this is B12 deficiency, not associated with sickle cell disease or Salmonella

A 23 year old female visits the local Urgent Care because of recent pain and swelling in her axillary region. The patient informs the physician of her high fever that preceded the painful bubo. The physician asks the patient about the scratch on her arm, which she explains was the result of trying to put the stray cat she adopted almost a week ago in the carrier for its veterinarian appointment for a flea bath. What virulence gene allows this bacteria to prevent opsonization and phagocytic migration, and spread rapidly throughout the host? A. virB operon B. Fraction 1 gene C. pInv genes D. Plasminogen activator protease gene E. Locus of enterocyte effacement pathogenicity island

Correct answer: D - the causative organism is Yersinia pestis, it has a plasminogen activator protease that degrades complement components (C3b & C5a) which prevent opsonization and phagocytic migration; this gene also degrades fibrin clots which allows it to spread rapidly throughout the host (Pages 261-262 under Yersinia) Incorrect - this encodes genes for virulence in Bartonella henselae, which causes cat scratch fever Incorrect - this gene is found in Y. pestis but codes for the antiphagocytic protein capsule, which prevents phagocytosis Incorrect - this gene is found in Enteroinvasive E. coli and allows for bacterial invasion into the colonic epithelium Correct! See above explanation Incorrect - this encodes genes for Enteropathogenic & Shiga-toxin producing E. coli and regulates the stages of attachment and is responsible for attachment and destruction of the host cell surface

A 26 year old male presents to the emergency clinic with complaints of fatigue, pale skin, dizziness, and jaundice. A bilirubin blood test was ordered and shows elevated levels of indirect bilirubin. The patient was diagnosed with hemolytic anemia causing a buildup of unconjugated bilirubin within the blood. Which of the following answers below causes the conjugation and eventual excretion of bilirubin? A. Glucose 6-phosphate dehydrogenase B. Glutathione peroxidase C. Thiamine pyrophosphate D. UDP-glucuronate transferase E. Gluconolactonase

Correct answer: D- Glucuronate residues are transferred from UDP-glucuronate to two carboxyl groups located on bilirubin causing the bilirubin to become conjugated. This transfer occurs through the use of UDP-glucuronate transferase, allowing for the bilirubin to be transported into bile for excretion, as the conjugated forms of bilirubin are much more soluble. A. Incorrect- oxidizes the aldehyde at carbon one of glucose 6-phosphate and reduces NADP+ to NADPH. This in turn also causes the formation of 6-phosphoglucono-δ-lactone. Involved in the conversion of glucose 6-phosphate into ribulose 5-phosphate. B. Incorrect- causes the conversion of hydrogen peroxide into 2 molecules of H2O. Glutathione peroxidase also causes the oxidation of glutathione, which is necessary for the removal of hydrogen peroxide and lipid peroxides generated from ROS. C. Incorrect- Coenzyme used with transketolase in the conversion of ribose-5-phosphate into glyceraldehyde 3-phosphate. D. Correct- see above E. Incorrect- Hydrolyzes 6-phosphoglucono-δ-lactone into 6-phosphogluconate, which is a sugar acid with carboxylic acid group at carbon 1. Involved in the conversion of glucose 6-phosphate into ribulose 5-phosphate.

A four-year-old male child presents to the pediatrician's office with his mother. The patient's mother states that her child has developed difficulty swallowing, impaired vision, and involuntary muscle spasms. Additionally, the patient's mother states that her child had several seizures in the past month. The doctor orders blood work which reveals mutation in the enzyme, called Hexosaminidase A (Hex A). The enzyme mutation is related to which of the following diseases? A. Fabry disease B. Krabbe disease C. Farber disease D. Tay-Sachs disease E. Niemann-Pick disease

Correct answer: D- Glycolipids are involved in intercellular communication. They are derivatives of the lipid sphingosine. These sphingolipids include the cerebrosides and the gangliosides. Defects in the degradation of sphingolipids l543-565ead to the sphingolipidoses also known as the gangliosidoses. Thus, this may lead to the mutation of Hexosaminidase A (Hex A) enzyme causing Tay-Sachs disease. A. Incorrect- caused by Alpha-Galactosidase enzyme deficiency B. Incorrect- caused by Beta-Galactosidase enzyme deficiency C. Incorrect- cased by Ceramidase enzyme deficiency D. Correct- see above E. Incorrect- caused by Sphingomyelinase enzyme deficiency

A 19-year-old African American male presents to his family physician with complaints of fatigue and weakness. He appears uncharacteristically pale. This is the second trip to his family physician this week. In his visit 3 days ago, his physician diagnosed him with viral hepatitis. While suffering from liver pain during that visit, he had not been feeling fatigued. His physician suspects a type of hemolytic anemia brought on by the environmental trigger of a viral infection. Which of the following when viewed on a blood smear would confirm the diagnosis of this specific hemolytic anemia? (A) Reticulocytes (B) Anisocytosis of Red Blood Cells (C) Howell-Jolly bodies (D) Bite Cells (E) Sickled Red Blood Cells

Correct answer: D- While other answer choices may be present in a blood smear of a Glucose-6-Phosphate Dehydrogenase patient, bite cells are unique to this disease. Oxidative stress in red blood cells causes the formation of membrane-bound precipitates known as Heinz bodies. As the RBC's enter the spleen for filtration, macrophages pluck out these Heinz bodies, leaving a partially devoured cell that appears to have a bite taken out. These cells are known as Bite Cells (A) Incorrect: While patients with hemolytic anemia have a higher proportion of reticulocytes than the average individual, this is not diagnostically specific to G6PD deficiency. (B) Incorrect: Anisocytosis of RBC's is common in many types of hemolytic anemia including Hereditary Spherocytosis and Thalassemia (C) Incorrect: This answer is common in Sickle Cell Anemia and Hereditary spherocytosis (D) Correct: See above (E) Incorrect: This answer is seen in Sickle Cell Anemia and not G6PD deficiency

A 17-year-old male arrives to the ER following a large car accident on the freeway. During the accident a piece of metal sliced into the patient's thigh, cutting their femoral artery in the process and causing a massive amount of blood loss. Which of the following changes are not expected to occur during the patient's bodily response to the blood loss? A. The patient will develop leukocytosis B. The patient's blood volume will initially be restored by water from the ISF C. The patient will have an increased reticulocyte count D. The patient's hematocrit will be lower than the average value E. The patient's blood volume will initially be restored by a rapid increase of bone marrow activity

Correct answer: E - An increase of bone marrow activity does result from acute blood loss. However, this is not how the body rapidly restores its blood volume immediately after acute blood loss, this is done by water leaving the ISF and entering the intravascular compartment, rather the increase in activity is mainly to restore the erythrocytes lost due to the acute blood loss. A. Incorrect - One would expect to see this because when acute blood loss occurs and blood pressure drops adrenergic hormones mobilize the granulocytes resulting in leukocytosis. B. Incorrect- One would expect to see this because when blood volume is lost the body immediately restores this volume through the movement of water. C. Incorrect - One would expect to see an increased reticulocyte (immature erythrocytes) count as the body works to replenish the lost erythrocytes. D. Incorrect - One would expect to see the hematocrit lower than average due to the loss of erythrocytes and this volume being replaced by water. E. Correct - See above.

A 27-year-old male presents to his PCP with complaints of exhaustion, dizziness, episodes of anxiety, and dark colored urine. He explains that his symptoms started 4 days ago but have been getting progressively worse. When his doctor asks for any changes in diet, the man explains that he and his friends have gone out to dinner twice in the past week to a Mediterranean restaurant where they shared a family sized chicken kabob pita, lentil soup, fava bean appetizer, and baklava. The doctor orders a blood smear and notes the presence of bite cells among normal red blood cells. What is the cause of the red blood cell abnormality and where does it occur? A. Decreased level of reduced glutathione causing a deformation of the membrane of a red blood cell in circulation B. Misfolded beta globin protein during formation in bone marrow C. Cross-linking of reactive sulfhydryl groups on globin chains while passing through pulmonary capillaries due to high levels of oxidants D. Hemolysis of RBCs in circulation via leukocytes E. Macrophages plucking Heinz bodies from red blood cells as they pass through the splenic cords

Correct answer: E - As red blood cells, containing Heinz bodies, pass through the splenic cords, macrophages pluck out the Heinz bodies. This process damages the membrane of red blood cells, giving them an abnormal shape that looks like a bite was taken out of the red blood cell. A. Incorrect - Decreased levels of reduced glutathione cause older red blood cells to become fragile and more susceptible to proteolytic degradation due to misfolding of the G6P protein. It also decreases deformability of RBCs, preventing them from squeezing through narrow capillaries or from carrying oxygen. RBCs do not take on the appearance of a bite cell. B. Incorrect - Misfolded beta globin protein during formation in bone marrow results in a sickle cell shape and inability to transport oxygen through the body C. Incorrect - Cross-linking of reactive sulfhydryl groups on globin chains causes the formation of Heinz bodies. Heinz bodies appear as dark inclusions in intact red blood cells. No bite appearance is seen D. Incorrect - hemolysis of RBCs results in the presence of schistocytes on a blood smear. Activated leukocytes (i.e. neutrophils) produce oxygen-derived free radicals during infections that damage fragile red blood cells E. Correct - see above

A Mediterranean patient presents to the emergency room with jaundice and discolored urine. Upon taking his history, it is found that he has recently eaten fava beans. The physician diagnosed the patient with G6PD deficiency. Which of the following occurs when this patient eats fava beans? a) Glutathione concentrations will increase b) Glutathione will become reduced c) Glycolysis will be inhibited d) Oxidative stress will decrease e) Hemoglobin will denature

Correct answer: E - Heinz bodies can be seen on the peripheral blood smear of a patient with G6PD deficiency. With this disorder, under oxidative stress such as eating fava beans, the lack of reduced glutathione will lead to damaged hemoglobin as seen in Heinz bodies. a) Incorrect - NADPH does inhibit G6PD, but not in G6PD deficient patients b) Incorrect - Glutathione concentrations are decreased in G6PD deficient patients c) Incorrect - G6PD deficiency does not affect glycolysis d) Incorrect - Oxidative stress will increase due to eating the fava beans e) Correct: see above

Last week, Michelle went out to Jack in the Box with her friends. Three days later, she began having watery diarrhea, which progressed to bloody diarrhea. Michelle is a naturally unlucky human being, but still decided to do absolutely nothing about her symptoms. Michelle starts feeling extreme fatigue and notices blood in her urine. Finally, she goes to the doctor where she is told that her EHEC infection has progressed to Hemolytic Uremic Syndrome (HUS). She is placed on supportive care. After the doctor examines Michelle's peripheral blood smear, she diagnoses her with Microangiopathic hemolytic anemia (MAHA). What did the doctor find on the peripheral blood smear that led her to this final diagnosis? A. Spherocytes B. Teardrop cells C. Pencil-shaped deformities D. Hypersegmented neutrophils E. Schistocytes

Correct answer: E - Microangiopathic hemolytic anemia (MAHA) is caused by the traumatic destruction of RBCs as they pass through small or damaged blood vessels. Schistocytes are fragmented erythrocytes - meaning they have been physically damaged. Another key clue was the diagnosis of Hemolytic Uremic Syndrome (HUS). HUS is one of the leading causes of MAHA with the other leading cause being thrombotic thrombocytopenic purpura (TTP). A. Spherocytes (incorrect) - Associated with immune hemolysis B. Teardrop cells (incorrect) - Associated with myelofibrosis C. Pencil-shaped deformities (incorrect) - Associated with severe iron deficiency D. Hypersegmented neutrophils (incorrect) - Associated with different types of megaloblastic anemia E. Schistocytes (CORRECT)

A 6 year old presents to the emergency room with her mother. The mother states her daughter started having watery diarrhea yesterday in addition to abdominal cramping and a fever. Both the mother and the daughter have never traveled outside of the U.S. and the daughter attends a daycare center after school. She is diagnosed with shigellosis. Based on the travel history, after school activities, and symptoms, which species of shigella is most likely causing the daughters symptoms? ' A. Shigella flexneri B. Shigella dysenteriae C. Shiga-toxin E. coli D. Shigella boydii E. Shigella sonnei

Correct answer: E - Shigella sonnei is the most common shigella species responsible for shigellosis in the U.S. Shigella sonnei causes approximately 85% of all shigella infections in the United states. A) Incorrect - Shigella flexneri causes majority of shigella infections in developing countries B) Incorrect - Shigella dysenteriae typically causes endemics in West Africa and Central America. Symptoms for shigella dysenteriae infections are more severe than Shigella sonnei commonly presenting with progression of watery diarrhea to high volumes of bloody diarrhea with mucus. C) Incorrect - Shiga-toxin E. coli is not a species of shigella. STEC is a strain of E. coli that picked up shiga-toxin from shigella. D) Incorrect - Shigella boydii infections are commonly associated with travel, and occur in developing countries. E) Correct - Shigella sonnei is the most common shigella species responsible for shigellosis in the U.S. Shigella sonnei causes approximately 85% of all shigella infections in the United states.

Describe the pathogenesis of encapsulated Enterobacteriaceae? A. protection from phagocytosis B. expression of alternating antigens C. transfer of bacterial virulence factors into targeted eukaryotic cell D. lipid A component of LPS E. produce siderophores

Encapsulated Enterobacteriaceae are protected from phago-cytosis by the hydrophilic capsular antigens, which repel the hydrophobic phagocytic cell surface. A. correct; see above explanation B. incorrect; this is the role of antigenic phase variation C. incorrect; this is the role of type III secretion systems D. incorrect; this is the role of endotoxins E. incorrect; this is the role of sequestration of growth factors

Ian is a 24-year-old professional cyclist who has qualified to compete in the 2024 Summer Olympics in Paris for Team USA. As part of the International Olympic Committee's Anti-doping campaign, Ian is chosen for a random blood test to check for the use of performance enhancing drugs. His blood test results are below: CBC component results Component Ian's Value Standard range Units WBC count 6.7 4.5 - 11.0 K/UL RBC count 13.8 4.7 - 6.1 MIL/UL Platelet count 221 150 - 420 K/UL MCV 89 80 - 95 fL MPV 8.8 7 - 10 fL Which of the following regulatory substances is responsible for regulating the formation and release of the blood component that is abnormal in Ian's CBC report? Which organ secretes this regulatory substance? a. Erythropoietin; thymus b. Thrombopoietin; kidneys c. Lymphopoietin; liver d. Erythropoietin; kidneys e. Reticulopoietin; bone marrow

Explanation: Correct Answer: D - Ian's CBC indicates an elevated RBC count. The regulatory substance that regulates RBC counts is erythropoietin and it is synthesized and secreted by the kidneys. A. Incorrect - Erythropoietin does regulate the formation and release of RBCs, but it is not secreted by the thymus B. Incorrect - thrombopoietin is a hormone that regulates production of platelets. Ian's platelet counts are normal so this cannot be the answer. C. Incorrect - lymphopoietin is not a hormone that regulates RBC counts. D. Correct - See above E. Incorrect - Reticulopoietin is not a real regulatory substance.

A 20 year old Hispanic man arrives at the Emergency Department with complaints of chest pain, fever, cough, and hypoxemia. The patient explains he is concerned that he has COVID-19 because his symptoms matched the symptoms listed on the COVID-19 checklist on WebMD. The patient is ordered a CT of his chest and is later found to have pulmonary infiltrates in bilateral lobes. Following the patient's chest CT and further lab results, the physician orders an emergency exchange transfusion due to the increasing respiratory compromise. Considering the patient's symptoms and CT findings, which complication of sickle cell disease was the patient diagnosed with? A. Thalassemia Intermedia B. Aplastic Crisis C. α-Thalassemia D. Acute Chest Syndrome E. Cooley's Anemia

Explanation: Most of the acute complications of sickle cell disease are a result of vaso-occlusion which causes ischemic injury to organs and severe pain. Patients often have painful crises that are precipitated by infections, dehydration, or even rapid changes in temperature. An example of vaso-occlusion in the pulmonary circulation is known as acute chest syndrome and is characterized by symptoms of chest pain, hypoxemia, and fever. Since further sickling and increasing respiratory compromise is a consequence of hypoxemia, acute chest syndrome is life-threatening and requires emergency exchange transfusion. A. Incorrect; Thalassemia intermedia consists of moderate hemolysis and severe anemia, but is not transfusion dependent B. Incorrect; Aplastic crisis is caused by any toxic or infectious insults that transiently suppress bone marrow activity C. Incorrect; α-Thalassemia consists of mild microcytic anemia D. Correct; acute chest syndrome is a complication of sickle cell disease and requires emergent exchange transfusion E. Incorrect; Cooley's anemia consists of severe hemolysis and ineffective erythropoiesis

A 68 year old man with Chronic Kidney disease, well known to his PCPs office, presents with some of his usual symptoms such as trouble sleeping, reduced exercise tolerance, and some angina. The physician does a blood test and finds decreased levels of iron. What is expected to be the patient's MCV value to be and what protein is causing his diminished iron levels? A. Decreased; hepcidin B. Normal; hepcidin C. Normal; EPO D. Elevated; EPO E. Elevated: hepcidin

Explanation: Normocytic anemia is present in many chronic conditions such as Chronic Kidney disease and exhibits normal MCV values (80-95/100 fL/cell). Hepcidin is the protein that reduces intestinal iron absorption/mobilization and is present in unusually high numbers in patients with chronic conditions associated with normocytic anemia. EPO is incorrect because it is of course Erythropoietin, a glycoprotein cytokine that stimulates RBC production. B

A wife came home and found her 57 year old husband passed out on the kitchen floor, thinking he was drinking again, she thought nothing of it and left him there. Hours later she sees her husband is still on the floor with pale skin, weakness, and he complains of diplopia. She takes him to the ER where the attending physician orders a CBC, oral examination and physical. The physician finds the patient has glossitis and cheilosis and his red blood cells appear swollen and ovoid. The physician then orders an MRI where neurological demyelination of the dorsal column is evident. Which of the following is most associated with the patient's prognosis? A) Increased rate of purine synthesis B) Increased secretions of intrinsic factor C) Increased bacterial/parasitic growth D) Decreased mean corpuscular volume E) Decreased fat soluble vitamin concentration

Explanation: The symptoms of pale skin, weakness, and diplopia can be thought to be related to the patient's drinking, however glossitis and cheilosis are common presentations in megaloblastic anemia and the appearance of large, oval cells (macroovalocytes) should help confirm this diagnosis. Furthermore, cobalamin deficiency can cause a range of neurological symptoms which is not seen in other causes of megaloblastic anemia. Out of the choices the only answer choice that could cause a cobalamin deficiency and therefore megaloblastic anemia is a bacterial infection or a tapeworm infection (parasite). A) Incorrect - because megaloblastic anemia is correlated with impared purine synthesis B) Incorrect - If a patient has a cobalamin deficiency they are lacking B12 and likely dont have large amounts of intrinsic factor for its absorption C) Correct - Both bacterial overgrowth and tapeworm infection can cause cobalamin deficiency D) Incorrect - Patients with megaloblastic anemia have increased MCV values since the RBC are abnormally large. Sideroblastic anemia would decrease MCV, however the presence of swollen ovoid RBC rule this choice out. E) Incorrect - The patient would have a decrease in water soluble (B12/B9) not fat soluble.

A 28 year old female presents to her Primary Care Physician for her annual physical. Upon talking with the patient, the physician finds it necessary to draw a blood test in order to determine levels of the patient's bodily iron stores. When the physician obtains the results of the blood tests, they determine that the levels of bodily iron stores are increased. What protein was elevated in the patient's blood that most likely led to this determination? A. Amylase B. Ferritin C. Hemoglobin D. Albumin E. Transferrin

The correct answer is Choice B: Ferritin. The amount of Ferritin in the blood is the most sensitive indicator of the body's iron stores. A. Amylase - incorrect due to amylase being the enzyme responsible for carbohydrate breakdown B. Ferritin - correct answer C. Hemoglobin - incorrect due to hemoglobin being the binding protein of oxygen in RBCs D. Albumin - incorrect due to albumin being a very common blood transport protein, but not an indicator of iron stores E. Transferrin - incorrect due to transferrin being a transporter of iron in the blood; however, not an indicator of iron stores in the body

A 43-year-old man returned from a recent visit with his parents. He goes to his primary care physician with complaints of lesions. He mentions hunting in the woods of South Carolina with his parents and receiving many bug bites. After some testing, the physician found that the bacteria causing the lesions has a prominent capsule and a mucoid appearance of the colonies. The man is diagnosed with Donovanosis. What other disease has similar resemblance to this diagnosis? A. Enteroaggregative escherichia coli B. Proteus mirabilis C. Rickettsia rickettsii D. Treponema pallidum E. Yersinia pestis

The man presents with lesions in an undescribed area due to a bacteria with a capsule and mucoid appearing colonies. The physician diagnosed the patient with Donovanosis, the historical name for Klebsiella granulomatis. This man has granuloma inguinale presenting with painless lesions. Nodules may appear and lead to lesions and the ulcers that resemble syphilitic lesions. A. Enteroaggregative escherichia coli: Incorrect. EAEC is a GI disease associated with chronic diarrhea, fever, and abdominal pain. B. Proteus mirabilis: Incorrect. Proteus are associated with UTI symptoms producing high amounts of urease leading to kidney stones. C. Rickettsia rickettsii: Incorrect. While the description of the patient's previous vacation describes a dog tick bite, his symptoms did not originate from the visit. K. granulomatis is uncommon in the US, however, incubation can take months to progress to syphilitic lesions. D. Treponema pallidum: Correct. See explanation above. E. Yersinia pestis: Incorrect. Y. pestis is normally transferred by rats and fleas. This disease is also associated with fever, malaise, and bubos (inflammation of lymph nodes in the groin/axilla areas).

A 35-year-old male chef initially presents to the clinic with nausea, vomiting, and anorexia. Upon further examination, the patient has a fever and complains of headache, myalgia, and malaise. The patient also mentions he recently traveled to Cambodia. The clinic cultured the organism on MacConkey agar and noticed the colonies grew colorless and were resistant to bile salts. The patient was educated about chronic colonization of the gallbladder and prescribed antibiotics. What is the organism responsible for causing the patient's symptoms? A. Salmonella Typhi B. Escherichia coli C. Klebsiella pneumoniae D. Salmonella Choleraesuis E. Citrobacter koseri

The patient shows signs of fever, headache, anorexia, and malaise. These are symptoms of typhoid fever, which is caused by Salmonella Typhi. This is confirmed by the combination of lab results which indicate that the organism is of the Salmonella genus. Salmonella Choleraesuis is not adapted to humans and does not cause typhoid fever. A. See above. B. Incorrect - Some Escherichia coli may be related to recent travels to developing countries, but E. coli is lactose fermenting, and their colonies are pink/purple on the MacConkey agar. C. Incorrect - Klebsiella pneumoniae is also a lactose fermenting organism. Their colonies will appear pinkish purple on the MacConkey agar. Klebsiella pneumoniae may cause lobar pneumonia, but the patient showed no signs and symptoms of pneumonia. D. Incorrect - Salmonella Choleraesuis is more adapted to animals and less likely to colonize the gallbladder of humans. E. Incorrect - Citrobacter koseri is a lactose fermenting microorganism. Their colonies on the MacConkey agar will appear pink/purple. Citrobacter koseri may cause meningitis and brain abscesses in neonates. The patient is not in the at-risk group and showed no signs or symptoms of meningitis or brain abscesses.

A family in the United States adopted a child from Africa at three months old. Now, three months later, the child is six months old, the family notices that their growth has stopped and their cheekbones have become large and malformed. A CBC shows that their HbF is elevated and the child is showing signs of hepatosplenomegaly. It is deduced that the child has some severe mutation/possible deletion in the gene of the synthesis of globin chains. What chains would be targeted in this condition? A. Alpha B. Beta C. Gamma D. Delta E. Zeta

This is a textbook presentation of Beta-thalassemia major, with the major key of deformed cheekbones. This condition occurs when there is a mutation of the beta-globin chains. Genotype may either appear as B0/B0, B+/B+ or B0/B+; all resulting in little to no beta globin. The condition would not have been detectable before 6 months of age, because that is when the HbF (in fetus) transferrs to HbA2 (adult) and the symptoms manifest. A. Incorrect: Alpha thalassemia occurs in many different forms, but none of them come with hepatosplenomegaly and deformed bony structures. Additionally, with this condition, one would definitely notice the presence of hemoglobin Barts in a blood smear. B. Correct: See above C. Incorrect: Gamma globin chains actually don't have their own form of thalassemia- you hear about gamma globin chains in the formation of hemoglobin Barts which is a hallmark of Alpha thalassemia. D. Incorrect: Delta globin chains' role in thalassemias is simply to, when unpaired, form insoluble inclusions within red cell precursors. E. Incorrect: Zeta chains are only found in Hydrops fetalis, which is a kind of alpha thalassemia.

A 24 year old patient presents to the ED with complaints of fatigue and pale appearance noticed by coworkers. They have been seen for this issue in the past, and indicates they have been diagnosed with "something that was passed down to me by my parents and makes my blood not right". The patient provides genetic testing results that reveal an autosomal dominant mutation of the membrane proteins spectrin, ankyrin, band 3, and protein 4.2. Upon further examination of the patient's medical history, they have noted splenomegaly seen by radiologists on previous imaging. What should be ordered to confirm this patient's diagnosis and what would be found on this test? A: Blood smear, acanthocytes B: Direct Coombs test, positive agglutination C: blood smear, microspherocytes and elliptocytes D: blood smear, prominent spherocytes E: Indirect Coombs test, positive agglutination

This patient has Hereditary spherocytosis, a type of hemolytic anemia. They state their condition was passed down by their parents, indicating that it is a hereditary condition, and the fact that it "makes their blood not right," along with their pale appearance, indicates anemia. An autosomal dominant mutation of membrane proteins spectrin, ankyrin, band 3, and protein 4.2 narrows down the inherited membrane abnormality to hereditary spherocytosis. In order to be sure and confirm this diagnosis, a blood smear should be ordered and would show prominent spherocytes. Hereditary spherocytosis is characterizes by hemolytic anemia, splenomegaly, and prominent spherocytes in peripheral blood. A is correct that the test ordered should be a blood smear, but you would see acanthocytes in spur cell anemia rather than HS B is incorrect in both parts: you would order a Coombs test for Immune hemolytic anemia rather than hemolytic anemia caused by disorders of the erythrocyte membrane. C is correct that the test ordered should be a blood smear, but microspherocytes and elliptocytes indicate hereditary pyropoikilocytosis rather than hereditary spherocytosis. E is incorrect in both parts: you would order a Coombs test for immune hemolytic anemia rather than hemolytic anemia caused by disorders of the erythrocyte membrane.

A 27-year-old female presents to the emergency room complaining of dysentery, severe stomach pain, and low-grade fever (99.5 F). She explains that she recently went to the cider mill with her roommate 3 days prior to her symptoms developing. She recalls drinking apple cider and having a few donuts while there. What is the causative agent, and which of the following agars would help differentiate what the causative agent is? A. EIEC, Eosin Methylene Blue agar B. ETEC, Blood agar C. STEC, MacConkey-Sorbitol agar D. STEC, Eosin Methylene Blue agar E. ETEC, MacConkey-Sorbitol agar

This patient has a history of drinking apple cider, which is a possible food source of STEC. She does not have a travel history, which would rule out both EIEC and ETEC. MacConkey-Sorbitol agar will help differentiate STEC, or O157:H7, from other E. coli that cause gastroenteritis. Unlike other strains of E.coli, O157:H7 does not ferment sorbitol, so it will appear colorless on S-MAC, as opposed to ETEC or EIEC, which will show up as pink colonies on S-MAC. A. Incorrect: This is not the correct causative agent. The patient does not have a travel history. A. Incorrect: This is not the correct causative agent. The patient does not have a travel history to South America, Caribbean, or Mexico. B. Correct. See above. C. Incorrect: This is the correct causative agent, however, this agar will not help differentiate STEC from other E. coli strains. D. Incorrect: This is not the correct causative agent. The patient does not have a travel history to South America, Caribbean, or Mexico.

A 32 year old woman has recently given birth to a seemingly healthy baby boy. 48 hours after delivery, the baby started to show signs of yellowing of the skin and sclerae of the eyes. A total bilirubin lab test shows elevated levels of total bilirubin and an indirect bilirubin lab test shows elevated levels of unconjugated bilirubin. The baby was diagnosed with neonatal jaundice. What is the most likely treatment for neonatal jaundice in this patient? A. Phototherapy B. Supportive therapy C. Sulfa antibiotic therapy D. No effective therapy is known E. Steroid therapy

● Answer from page 1210 (PDF version) ● Many full term newborns develop neonatal jaundice. It is usually caused by an increased destruction of RBCs after birth and an immature bilirubin conjugating system in the liver (especially if the fetus had an unusually large number of RBCs). This leads to elevated levels of unconjugated bilirubin deposited in hydrophobic environments. If these levels reach a certain number at the age of 48 hours, the newborn is a candidate for phototherapy. The child is placed under lamps or on a "biliblanket" that emits light and bilirubin absorbs the light (undergoes chemical changes and becomes more water-soluble). Within a week of birth, the liver should be able to handle RBC turnover rates. A. Phototherapy - correct - please see the above explanation. B. Supportive therapy - incorrect - there is a potential treatment for this condition in this patient because the patient is 48 hours old, so supportive therapy would not be the best answer. C. Sulfa antibiotic therapy - incorrect - as mentioned in this chapter, certain drugs (such as sulfa drugs and antimalarials), infectious agents, and exposure to fava beans can actually lead to RBC destruction and jaundice in individuals (with a genetic deficiency of glucose 6-P dehydrogenase). D No effective therapy is known - incorrect - there is no known effective therapy for Tay Sach's disease, as mentioned in this chapter, but phototherapy can help babies with neonatal jaundice if they are 48 hours old. E Steroid therapy - incorrect - like bilirubin, xenobiotics, drugs, steroids, and other compounds with hydroxyl groups and low solubility in water are converted to glucuronides by glucuronyltransferases present in the endoplasmic reticulum and cytoplasm of liver and kidney (glucuronate increases solubility of the compound in water by adding negative charges in order to excrete it). So, steroids would not be a potential therapy for this condition. Rather, they use a similar conjugation pathway as bilirubin.


Related study sets

Neurologic and Cognitive Function Quiz: Exam 4

View Set

Lippincott chapter 1 health promotion care of a child missed questions

View Set

Psychology 6.1 What Is Learning?

View Set

Assessment & Care of Patients with Fluid & Electrolyte Imbalances

View Set

13th- Documentary Vocabulary Assignment.

View Set

Lesson 4: Real Estate Brokerage and Law of Agency/Ethics

View Set